*NURSING > QUESTIONS & ANSWERS > AHIMA CCA: All Top Exam Questions with Accurate answers, Test Bank. 100% Accurate, Verified. Graded (All)

AHIMA CCA: All Top Exam Questions with Accurate answers, Test Bank. 100% Accurate, Verified. Graded A+ 1.. TESTBANK.

Document Content and Description Below

AHIMA CCA: All Top Exam Questions with Accurate answers, Test Bank. 100% Accurate, Verified. Graded A+ 1. A 65-year-old white male was admitted to the hospital on 1/15 complaining of abdominal pa... in. The attending physician requested an upper GI series and laboratory evaluation of CBC and UA. The x-ray revealed possible cholelithiasis, and the UA showed an increased white blood cell count. The patient was taken to surgery for an exploratory laparoscopy, and a ruptured appendix was discovered. The chief complaint was: a. Ruptured appendix b. Exploratory laparoscopy c. Abdominal pain d. Cholelithiasis - ✔✔c. Abdominal pain The nature and duration of the symptoms that caused the patient to seek medical attention as stated in the patient's own words (Odom-Wesley et al. 2009, 331). 2. 84. An individual stole and used another person's insurance information to obtain medical care. This action would be considered: a. Violation of bioethics b. Fraud and abuse c. Medical identity theft d. Abuse - ✔✔c. Medical identity theft Correct Answer: 84. c. Medical identity theft occurs when someone uses a person's name and sometimes other parts of their identity without the victim's knowledge or consent to obtain medical services or goods (Johns 2011, 773). 3. Identify the ICD-9-CM diagnostic code(s) for acute osteomyelitis of ankle due to Staphylococcus. a. 730.06 b. 730.07 c. 730.07, 041.1 d. 730.07, 041.10 - ✔✔d. 730.07, 041.10 Correct Answer: D Index Osteomyelitis, acute or subacute. Refer to the table in the Index for the fifth digit 5, ankle and foot. Infection, staphylococcal NEC (Schraffenberger 2012, 305-306). 4. A system that provides alerts and reminders to clinicians is a(n): a. Clinical decision support system b. Electronic data interchange c. Point of care charting system d. Knowledge database - ✔✔Correct Answer: A Clinical decision support includes providing documentation of clinical findings and procedures, active reminders about medication administration, suggestions for prescribing less expensive but equally effective drugs, protocols for certain health maintenance procedures, alerts that a duplicate lab test is being ordered, and countless other decisionmaking aids for all stakeholders in the care process (Johns 2011, 138). 5. What does an audit trail check for? a. Unauthorized access to a system b. Loss of data c. Presence of a virus d. Successful completion of a backup - ✔✔Correct Answer: A Audit trails can provide tracking information such as who accessed which records and for what purpose (Johns 2011, 403). 6. This is a condition with an imprecise diagnosis with various characteristics. The condition may be diagnosed when a patient presents with sinus arrest, sinoatrial exit block, or persistent sinus bradycardia. This syndrome is often the result of drug therapy, such as digitalis, calcium channel blockers, beta-blockers, sympatholytic agents, or antiarrhythmics. Another presentation includes recurrent supraventricular tachycardias associated with bradyarrhythmias. Prolonged ambulatory monitoring may be indicated to establish a diagnosis of this condition. Treatment includes insertion of a permanent cardiac pacemaker. a. Atrial fibrillation (427.31) b. Atrial flutter (427.32) c. Paroxysmal supraventricular tachycardia (427.0) d. Sick sinus syndrome (SSS) (427.81) - ✔✔d. Sick sinus syndrome (SSS) (427.81) Correct Answer: D SSS is the imprecise diagnosis with various characteristics treated with the insertion of a permanent cardiac pacemaker. The other three conditions are treated with cardioversion and different pharmacological therapy (Schraffenberger 2012, 194-195). 7. Identify the ICD-9-CM diagnostic code for primary localized osteoarthrosis of the hip. a. 715.95 b. 715.15 c. 721.90 d. 715.16 - ✔✔b. 715.15 Correct Answer: B Index Osteoarthrosis, localized, primary. For category 715, refer to the table for the fifth digit of 5 for pelvic region and thigh (Schraffenberger 2012, 303-304). 8. A health record with deficiencies that is not complete within the timeframe specified in the medical staff rules and regulations is called a(n): a. Suspended record b. Delinquent record c. Pending record d. Illegal record - ✔✔Correct Answer: B An incomplete record not rectified within a specific number of days as indicated in the medical staff rules and regulations is considered to be delinquent (Johns 2011, 412). 9. A hospital HIM department wants to purchase an electronic system that records the location of health records removed from the filing system and documents the date of their return to the HIM department. Which of the following electronic systems would fulfill this purpose? a. Chart deficiency system b. Chart tracking system c. Chart abstracting system d. Chart encoder - ✔✔Correct Answer: B With an automated tracking system, it is easy to track how many records are charged out of the system, their location, and whether they have been returned on the due dates indicated (Johns 2011, 402). 10. Identify the appropriate ICD-9-CM diagnosis code for Lou Gehrig's disease. a. 335.20 b. 334.8 c. 335.29 d. 335.2 - ✔✔Correct Answer: A Index Disease, Lou Gehrig's or Lou Gehrig's disease. Amyotrophic lateral sclerosis is another name for Lou Gehrig's disease. Many diseases carry the name of a person or an eponym. The main terms for eponyms are located in the Alphabetic Index under the eponym or the disease, syndrome, or disorder (Schraffenberger 2012, 13). 11. In the laboratory section of CPT, if a group of tests overlaps two or more panels, report the panel that incorporates the greatest number of tests to fulfill the code definition. What would a coder do with the remaining test codes that are not part of a panel? a. Report the remaining tests using individual test codes, according to CPT. b. Do not report the remaining individual test codes. c. Report only those test codes that are part of a panel. d. Do not report a test code more than once regardless whether the test was performed twice. - ✔✔Correct Answer: A Reporting additional test codes that overlap codes in a panel allows the coder to assign all appropriate codes for services provided. It is inappropriate to assign additional panel codes when all codes in the panel are not performed. Reporting individual lab codes is appropriate when all codes in a panel have not been provided (AMA 2012b, 402). 12. An electrolyte panel (80051) in the laboratory section of CPT consists of tests for carbon dioxide (82374), chloride (82435), potassium (84132), and sodium (84295). If each of the component codes are reported and billed individually on a claim form, this would be a form of: a. Optimizing b. Unbundling c. Sequencing d. Classifying - ✔✔Correct Answer: B Unbundling occurs when a panel code exists and the individual tests are reported rather than the panel code (AMA 2012b, 402). 13. Coronary arteriography serves as a diagnostic tool in detecting obstruction within the coronary arteries. Identify the technique using two catheters inserted percutaneously through the femoral artery. a. Combined right and left (88.54) b. Stones (88.55) c. Judkins (88.56) d. Other and unspecified (88.57) - ✔✔Correct Answer: C The Judkins technique provides x-ray imaging of the coronary arteries by introducing one catheter into the femoral artery with maneuvering up into the left coronary artery orifice, followed by a second catheter guided up into the right coronary artery, and subsequent injection of a contrast material (Schraffenberger 2012, 206). Ensuring the continuity of future care by providing information to the patient's attending physician, referring physician, and any consulting physicians is a function of the: a. Discharge summary b. Autopsy report c. Incident report d. Consent to treatment - ✔✔Correct Answer: A The discharge summary is a concise account of the patient's illness, course of treatment, response to treatment, and condition at the time the patient is discharged (Johns 2011, 78). 15. This document includes a microscopic description of tissue excised during surgery: a. Recovery room record b. Pathology report c. Operative report d. Discharge summary - ✔✔Correct Answer: B The pathology report describes specimens examined by the pathologist (Johns 2011, 77). 16. CMS developed medically unlikely edits (MUEs) to prevent providers from billing units of services greater than the norm would indicate. These MUEs were implemented on January 1, 2007, and are applied to which code set? a. Diagnosis-related groups b. HCPCS/CPT codes c. ICD-9-CM diagnosis and procedure codes d. Resource utilization groups - ✔✔Correct Answer: B CMS developed MUEs to prevent providers from billing units in excess and receiving inappropriate payments. This new editing was the result of the outpatient prospective payment system which pays providers passed on the HCPCS/CPT code and units. Payment is directly related to units for specified HCPCS/CPT codes assigned to an ambulatory payment classification (CMS 2012b). 17. Identify the ICD-9-CM diagnostic code for other specified aplastic anemia secondary to chemotherapy. a. 284.9 b. 284.89 c. 285.9 d. 285.22 - ✔✔Correct Answer: B Index Anemia, aplastic, due to, antineoplastic chemotherapy. A coder should always assign the most specific type of anemia. Anemia due to chemotherapy is often aplastic (Schraffenberger 2012, 133-135 ). 18. When the physician does not specify the method used to remove a lesion during an endoscopy, what is the appropriate procedure? a. Assign the removal by snare technique code. b. Assign the removal by hot biopsy forceps code. c. Assign the ablation code. d. Query the physician as to the method used. - ✔✔Correct Answer: D It is not appropriate for the coder to assume the removal was done by either snare or hot biopsy forceps. The ablation code is only assigned when a lesion is completely destroyed and no specimen is retrieved. The coding professional must query the physician to assign the appropriate code (AHIMA 2012a, 607). 19. What is the best reference tool to determine how CPT codes should be assigned? a. Local coverage determination from Medicare b. American Medical Association's CPT Assistant newsletter c. American Hospital Association's Coding Clinic d. CMS website - ✔✔Correct Answer: B CPT Assistant provides additional CPT coding guidance on how to assign a CPT code by providing intent on the use of the code and explanation of parenthetical instructions. The American Medical Association publishes the guidance monthly (AMA 2012b). 20. Identify the appropriate ICD-9-CM diagnosis code(s) for right and left bundle branch block. a. 426.3, 426.4 b. 426.53 c. 426.4, 426.53 d. 426.52 - ✔✔Correct Answer: B Index Block, left, with right bundle branch block. Right and left bundle branch block is inclusive of one code. It is inappropriate to assign a code for right (426.4) and left (426.3) bundle branch block when a combination code includes both the right and left (Schraffenberger 2012, 201-207). 21. A software interface is a: a. Device to enter data b. Protocol for describing data c. Program to exchange data d. Standard vocabulary - ✔✔Correct Answer: C A software interface is a computer program that allows different applications to communicate and exchange data (Johns 2011, 137). 22. What did the Centers of Medicare and Medicaid Services develop to promote national correct coding methodologies and to control improper coding leading to inappropriate payment in Part B claims? a. Outpatient Perspective Payment System (OPPS) b. National Correct Coding Initiative (NCCI) c. Ambulatory Payment Classifications (APCs) d. Comprehensive Outpatient Rehab Facilities (CORFs) - ✔✔Correct Answer: B CMS developed the NCCI to control improper coding practices leading to inappropriate payments in Part B claims (CMS 2012a). 23. Identify the appropriate diagnostic and/or procedure ICD-9-CM code(s) for reprogramming of a cardiac pacemaker. a. V53.31 b. 37.85 c. V53.02 d. V53.31, 37.85 - ✔✔Correct Answer: A Index Fitting (of) pacemaker (cardiac). No procedure code exists in ICD-9-CM to describe reprogramming (Schraffenberger 2012, 204-205). 24. Which of the following hospitals are excluded from the Medicare acute-care prospective payment system? a. Children's b. Small community c. Tertiary d. Trauma - ✔✔Correct Answer: A Children's hospitals are excluded from PPS because the PPS diagnosis-related groups do not accurately account for the resource costs for the types of patients treated (Johns 2011, 321). 25. Which of the following programs has been in place in hospitals for years and has been required by the Medicare and Medicaid programs and accreditation standards? a. Internal DRG audits b. Peer review c. Managed care d. Quality improvement - ✔✔Correct Answer: D Quality improvement (QI) programs have been in place in hospitals for years and have been required by the Medicare/Medicaid programs and accreditation standards. QI programs have covered medical staff as well as nursing and other departments or processes (LaTour and Eichenwald Maki 2010, 33). 26. Identify the code for a patient with a closed transcervical fracture of the epiphysis. a. 820.09 b. 820.02 c. 820.03 d. 820.01 - ✔✔Correct Answer: D Index Fracture, femur, epiphysis, capital. Fifth digits are required for further classification of a specific condition. Many publishers include special symbols and/or color highlighting to identify codes that require a fourth or fifth digit (Schraffenberger 2012, 7). 27. What is the best source of documentation to determine the size of a removed malignant lesion? a. Pathology report b. Post-acute care unit record c. Operative report d. Physical examination - ✔✔Correct Answer: C The total size of a removed lesion, including margins, is needed for accurate coding. This information is best provided in the operative report. The pathology report typically provides the specimen size rather than the size of the excised lesion. Because the specimen tends to shrink, this is not an accurate measurement (Kuehn 2012, 110-111). 28. Which of the following definitions best describes the concept of confidentiality? a. The right of individuals to control access to their personal health information b. The protection of healthcare information from damage, loss, and unauthorized alteration c. The expectation that personal information shared by an individual with a healthcare provider during the course of care will be used only for its intended purpose d. The expectation that only individuals with the appropriate authority will be allowed to access healthcare information - ✔✔Correct Answer: C Confidentiality refers to the expectation that the personal information shared by an individual with a healthcare provider during the course of care will be used only for its intended purpose (Johns 2011, 49). 29. Identify the ICD-9-CM diagnosis code for Paget's disease of the bone (no bone tumor noted). a. 170.9 b. 213.9 c. 238.0 d. 731.0 - ✔✔Correct Answer: D Index Paget's disease, bone. The main terms for eponyms are located in the Alphabetic Index under the eponym or the disease, syndrome, or disorder (Schraffenberger 2012, 13). 30. Which of the following fails to meet the CMS classification of a hospital-acquired condition? a. Foreign object retained after surgery b. Air embolism c. Gram-negative pneumonia d. Blood incompatibility - ✔✔Correct Answer: C Gram-negative pneumonia (Johns 2011, 326). 31. Which of the following is (are) the correct ICD-9-CM procedure code(s) for cystoscopy with biopsy? a. 57.34 b. 57.32, 57.33 c. 57.33 d. 57.39 - ✔✔Correct Answer: C Index Cystoscopy (transurethral), with biopsy (Schraffenberger 2012, 251). 32. Identify the ICD-9-CM diagnosis code for chondromalacia of the patella. a. 717.7 b. 733.92 c. 748.3 d. 716.86 - ✔✔Correct Answer: A Index Chondromalacia, patella (Schraffenberger 2012, 303-304). 33. Identify the ICD-9-CM diagnosis code for blighted ovum. a. 236.1 b. 661.00 c. 631.8 d. 634.90 - ✔✔Correct Answer: C Index Ovum, blighted (Schraffenberger 2012, 282-283). 34. Each year the OIG develops a work plan that details areas of compliance it will be investigating for that year. What is the expectation of the hospital in relation to the OIG work plan? a. Hospitals are required to follow the same work plan and deploy audits based on that work plan. b. Hospitals should plan their compliance and auditing projects around the OIG work plan to ensure they are in compliance with the target areas in the plan. c. Hospitals must not develop their audits based on the OIG work plan; rather, they must develop their own and look for high-risk areas that need improvement. d. Hospitals must use the plan developed by their state hospital association that is specific to state laws and compliance activities. - ✔✔Correct Answer: B Hospitals are encouraged but not required to follow the same work plan as the OIG. Hospitals should review the plan carefully and plan their compliance program around the target areas (Johns 2011, 275). 35. The _____ may contain information about diseases among relatives in which heredity may play a role. a. Physical examination b. History c. Laboratory report d. Administrative data - ✔✔Correct Answer: B A complete medical history documents the patient's current complaints and symptoms and lists the patient's past medical, social, and family history (Johns 2011, 63). 36. There are several codes to describe a colonoscopy. CPT code 45378 describes the most basic colonoscopy without additional services. Additional codes in the colonoscopy section of CPT further define removal of foreign body (45379); biopsy, single or multiple (45380); and others. Reporting the basic form of a colonoscopy (45378) with a foreign body (45379) or biopsy code (45380) would violate which rule? a. Unbundling b. Optimizing c. Sequencing d. Maximizing - ✔✔Correct Answer: A The coder should assign the most comprehensive code to describe the entire procedure performed. When a code describes the entire service provided, the coder should not code each component separately. Assigning additional codes inherent to the main code would be a form of unbundling (Hazelwood and Venable 2012, 336). 37. Corporate compliance programs were released by the OIG for hospitals to develop and implement their own compliance programs. All of the following except _____ are basic elements of a corporate compliance program. a. Designation of a Chief Compliance Officer b. Implementation of regular and effective education and training programs for all employees c. Medical staff appointee for documentation compliance d. The use of audits or other evaluation techniques to monitor compliance - ✔✔Correct Answer: C Seven elements are required as part of the basic elements of a corporate compliance program and a medical staff appointee is not one of them (Johns 2011, 274). 38. The electronic claim format (837I) replaces which paper billing form? a. CMS-1500 b. CMS-1450 (UB-04) c. UB-92 d. CMS-1400 - ✔✔Correct Answer: B The electronic claim form (screen 837I) replaced the UB-04 (CMS 1450) paper billing form (Johns 2011, 343). 39. According to the Joint Commission Accreditation Standards, which document must be placed in the patient's record before a surgical procedure may be performed? a. Admission record b. Physician's order c. Report of history and physical examination d. Discharge summary - ✔✔Correct Answer: C According to the Joint Commission, except in emergency situations, every surgical patient's chart must include a report of a complete history and physical conducted no more than seven days before the surgery is to be performed (Odom-Wesley et al. 2009, 150). 40. The right of an individual to keep information about himself or herself from being disclosed to anyone is a definition of: a. Confidentiality b. Privacy c. Integrity d. Security - ✔✔Correct Answer: B Privacy is the right of an individual to be left alone. It includes freedom from observation or intrusion into one's private affairs and the right to maintain control over certain personal and health information (Johns 2011, 755). 41. Standardizing medical terminology to avoid differences in naming various medical conditions and procedures (such as the synonyms bunionectomy, McBride procedure, and repair of hallus valgus) is one purpose of: a. Transaction standards b. Content and structure standards c. Vocabulary standards d. Security standards - ✔✔Correct Answer: C Vocabulary standards establish common definitions for medical terms to encourage consistent descriptions of an individual's condition in the health record (Johns 2011, 227). 42. An outpatient clinic is reviewing the functionality of a computer system it is considering purchasing. Which of the following datasets should the clinic consult to ensure all the federally required data elements for Medicare and Medicaid outpatient clinical encounters are collected by the system? a. DEEDS b. EMEDS c. UACDS d. UHDDS - ✔✔Correct Answer: C Uniform Ambulatory Care Data Set (Odom-Wesley et al. 2009, 310). 43. Identify the ICD-9-CM diagnostic code for diastolic dysfunction. a. 428.1 b. 428.30 c. 428.9 d. 429.9 - ✔✔Correct Answer: D Index Dysfunction, diastolic (Schraffenberger 2012, 182-183). 44. Identify the appropriate ICD-9-CM procedure code(s) for a double internal mammary-coronary artery bypass. a. 36.15, 36.16 b. 36.15 c. 36.16 d. 36.12, 36.16 - ✔✔Correct Answer: C Index Bypass, internal mammary-coronary artery (single), double vessel (36.16). Internal mammarycoronary artery bypass is accomplished by loosening the internal mammary artery from its normal position and using the internal mammary artery to bring blood from the subclavian artery to the occluded coronary artery. Codes are selected based on whether one or both internal mammary arteries are used, regardless of the number of coronary arteries involved (Schraffenberger 2012, 203-204). 45. Identify the CPT code(s) for the following patient: A 2-year-old male presented to the emergency room in the middle of the night to have his nasogastric feeding tube repositioned through the duodenum under fluoroscopic guidance. a. 43752 b. 43761 c. 43761, 76000 d. 49450 - ✔✔Correct Answer: C Code 43761 is assigned to report repositioning of a nasogastric or orogastric feeding tube through the duodenum. An instructional note guides the coder to report code 76000 when image guidance is performed (AMA 2012b, 235). Which of the following is the correct ICD-9-CM procedure code for a Mayo operation known as a bunionectomy? a. 77.54 b. 77.69 c. 77.59 d. 77.51 - ✔✔Correct Answer: C Index Bunionectomy or Mayo operation, bunionectomy. The main terms for eponyms are located in the Alphabetic Index under the eponym or the disease, syndrome, operation, or disorder (Schraffenberger 2012, 13). 47. Whereas the focus of inpatient data collection is on the principal diagnosis, the focus of outpatient data collection is on: a. Reason for admission b. Reason for encounter c. Discharge diagnosis d. Activities of daily living - ✔✔Correct Answer: B The Uniform Ambulatory Care Data Set (UACDS) includes data elements specific to ambulatory care, such as the reason for the encounter with the healthcare provider (LaTour and Eichenwald Maki 2010, 166). 48. How do accreditation organizations such as the Joint Commission use the health record? a. To serve as a source for case study information b. To determine whether the documentation supports the provider's claim for reimbursement c. To provide healthcare services d. To determine whether standards of care are being met - ✔✔Correct Answer: D Surveyors review the documentation of patient care services to determine whether the standards for care are being met (Johns 2011, 40). 49. Mildred Smith was admitted from an acute-care hospital to a nursing facility with the following information: "Patient is being admitted for organic brain syndrome." Underneath the diagnosis, her medical information along with her rehabilitation potential were also listed. On which form is this information documented? a. Transfer or referral b. Release of information c. Patient rights acknowledgement d. Admitting physical evaluation - ✔✔Correct Answer: A The transfer or referral form provides document communication between caregivers in multiple healthcare settings. It is important that a patient's treatment plan be consistent as the patient moves through the healthcare delivery system (Odom-Wesley et al. 2009, 131). 50. Which of the following statements is true? a. The higher the relative weight, the higher the payment rates. b. The lower the relative weight, the higher the payment rates. c. The lower the relative weight, the sicker the patient. d. The higher the relative weight, the lesser reimbursement due the facility. - ✔✔Correct Answer: A Higher relative weights link to higher payment rates (Casto and Layman 2011, 13). 51. A coder needs to locate electronic health records for a patient across a health information exchange (HIE). What tool(s) should the coder use? a. Certification b. Identity-matching algorithm and record locator service c. Interoperability and certification d. Meaningful use - ✔✔Correct Answer: B An HIE organization requires an identity-matching algorithm and record locator service (RLS). An identity-matching algorithm must be used by the HIE to identify any patient for whom data are to be exchanged. This algorithm uses sophisticated probability equations to identify patients. The RLS, then, is a process that seeks information about where a patient may have a health record available to the HIE organization (Johns 2011, 151). 52. All documentation entered in the medical record relating to the patient's diagnosis and treatment is considered this type of data: a. Clinical b. Identification c. Secondary d. Financial - ✔✔Correct Answer: A Clinical information is data related to the patient's diagnosis or treatment in a healthcare facility (Odom-Wesley et al. 2009, 55). 53. What type of data is exemplified by the insured party's member identification number? a. Demographic data b. Clinical data c. Certification data d. Financial data - ✔✔Correct Answer: D Financial data include details about the patient's occupation, employer, and insurance coverage (OdomWesley et al. 2009, 42). 54. What is the best reference tool for ICD-9-CM coding advice? a. AMA's CPT Assistant b. AHA's Coding Clinic for HCPCS c. AHA's Coding Clinic for ICD-9-CM d. National Correct Coding Initiative (NCCI) - ✔✔Correct Answer: C AHA's Coding Clinic for ICD-9-CM is a quarterly publication of the Central Office on ICD-9-CM, which allows coders to submit a request for coding advice through the coding publication. 55. Identify the ICD-9-CM diagnostic code(s) for the following: A 6-month-old child is scheduled for a clinic visit for a routine well child exam. The physician documents, "well child, expreemie." a. V20.1, 765.10 b. V20.2 c. V20.2, 765.10 d. V20.2, 765.19 - ✔✔Correct Answer: C Index Exam, well baby. Premature, infant NEC. Refer to table in Tabular for fifth digit of "0" to note unspecified birth weight (Schraffenberger 2012, 324-328, ). 56. Identify the ICD-9-CM diagnostic code(s) and procedure code(s) for the following: term pregnancy with failure of cervical dilation; lower uterine segment cesarean delivery with single liveborn female. a. 661.01, V27.0, 74.1 b. 661.21, 74.1 c. 661.01, 74.0 d. 661.21, V27, 74.1 - ✔✔Correct Answer: A Index Delivery, cesarean, poor dilation, cervix (661.0). Refer to the ICD-9-CM Tabular (660-669) for the correct fifth digit of "1," delivered, with or without mention of antepartum condition. Outcome of delivery, single, liveborn. Cesarean section, low uterine segment (Schraffenberger 2012, 282-283). 57. A hospital receives a valid request from a patient for copies of her medical records. The HIM clerk who is preparing the records removes copies of the patient's records from another hospital where the patient was previously treated. According to HIPAA regulations, was this action correct? a. Yes; HIPAA only requires that current records be produced for the patient. b. Yes; this is hospital policy over which HIPAA has no control. c. No; the records from the previous hospital are considered part of the designated record set and should be given to the patient. d. No; the records from the previous hospital are not included in the designated record set but should be released anyway. - ✔✔Correct Answer: C The designated record set includes health records that are used to make decisions about the individual (Johns 2011, 822). 58. As recommended by AHIMA, HIM compliance policies and procedures should ensure all of the following except: a. Compensation for coders and consultants does not provide any financial incentive to code claims improperly b. The proper selection and sequencing of diagnoses codes c. Proper and timely documentation obtained prior to and after billing d. d The correct application of official coding rules and guidelines - ✔✔Correct Answer: C Proper and timely documentation of all physician and other professional services must be obtained prior to billing. Facilities should not provide any financial incentive that may tempt a coder to code claims improperly such as upcoding to higher DRGs, which result in higher pay (Johns 20011, 275). 59. What reimbursement system uses the Medicare fee schedule? a. APCs b. MS-DRGs c. RBRVS d. RUG-III - ✔✔Correct Answer: C The resource-based relative value scale (RBRVS) system was implemented by CMS in 1992 for physicians' services such as office visits covered under Medicare Part B. The system reimburses physicians according to a fee schedule based on predetermined values assigned to specific services (Johns 2011, 326). 60. The CIA of security includes confidentiality, data integrity, and data _____. a. Accessibility b. Authentication c. Accuracy d. Availability - ✔✔orrect Answer: D Security measures not only provide for confidentiality, but data integrity and data availability—the CIA of security (Johns 2011, 184). 61. Valley High, a skilled nursing facility, wants to become certified to take part in federal government reimbursement programs such as Medicare. What standards must the facility meet in order to become certified for these programs? a. Joint Commission Accreditation Standards b. Accreditation Association for Ambulatory Healthcare Standards c. Conditions of Participation d. Outcomes and Assessment Information Set - ✔✔Correct Answer: C Participating organizations must follow the Medicare Conditions of Participation to receive federal funds from the Medicare program for services rendered (Johns 2011, 61). 62. What software will prompt the user through a variety of questions and choices based on the clinical terminology entered to assist the coder in selecting the most appropriate code? a. Logic-based encoder b. Automated code book c. Speech recognition d. Natural-language processing - ✔✔Correct Answer: A Encoders come in two distinct categories: logic-based and automated codebook formats. A logic-based encoder prompts the user through a variety of questions and choices based on the clinical terminology entered. The coder selects the most accurate code for a service or condition (and any possible complications or comorbidities). An automated codebook provides screen views that resemble the actual format of the coding system (LaTour and Eichenwald Maki 2010, 269). 63. Which of the following is not a function of the discharge summary? a. Providing information about the patient's insurance coverage b. Ensuring the continuity of future care c. Providing information to support the activities of the medical staff review committee d. Providing concise information that can be used to answer information requests - ✔✔Correct Answer: A The discharge summary provides an overview of the entire medical encounter to ensure the continuity of future care by providing information to the patient's attending physician, referring physician, and any consulting physicians, to provide information to support the activities of the medical staff review committee and to provide concise information that can be used to answer information requests from authorized individuals or entities (Johns 2011, 78). 64. MS diagnostic-related groups are organized into: a. Case-mix classifications b. Geographic practice cost indices c. Major diagnostic categories d. Resource-based relative values - ✔✔Correct Answer: C Major diagnostic categories (MDCs), of which there are 25. The principal diagnosis determines the MDC assignment (Johns 2011, 322). 65. Identify ICD-9-CM diagnosis code for atypical ductal hyperplasia. a. 610.1 b. 610.4 c. 610.8 d. 610.9 - ✔✔Correct Answer: C Use this code when the diagnosis is specified as a certain type of "benign mammary dysplasia," and in this case, "ductal" hyperplasia. Index Hyperplasia, breast, ductal, atypical (Schraffenberger 2012, 253). 66. The Medical Record Committee is reviewing the privacy policies for a large outpatient clinic. One of the members of the committee remarks that he feels the clinic's practice of calling out a patient's full name in the waiting room is not in compliance with HIPAA regulations and that only the patient's first name should be used. Other committee members disagree with this assessment. What should the HIM director advise the committee? a. HIPAA does not allow a patient's name to be announced in a waiting room. b. There is no HIPAA violation for announcing a patient's name, but the committee may want to consider implementing practices that might reduce this practice. c. HIPAA allows only the use of the patient's first name. d. HIPAA requires that patients be given numbers and only the number be announced. - ✔✔Correct Answer: B It is suggested that covered entities use PHI with certain specified direct identifiers removed as a guideline for disclosing only minimum necessary information while providing the amount needed to accomplish the intended purpose (Johns 2011, 822). 67. CMS identified conditions that are not present on admission and could be "reasonably preventable," and therefore hospitals are not allowed to receive additional payment for these conditions that do present. What are these conditions called? a. a Conditions of Participation b. Present on admission c. Hospital-acquired conditions d. Hospital-acquired infection - ✔✔Correct Answer: C CMS identified hospital-acquired conditions (not present on admission) as "reasonably preventable," and hospitals do not receive additional payment for cases in which these cases are present (Johns 2011, 326). 68. Which of the following is (are) the correct ICD-9-CM code(s) for laparoscopic cholecystectomy? a. 51.21 b. 51.22, 54.21 c. 51.23, 54.21 d. 51.23 - ✔✔Correct Answer: D Index Cholecystectomy (total), laparoscopic (Schraffenberger 2012, 237-238). 69. The HIM manager is concerned about whether the data transmitted across the hospital network is altered during the transmission. The concept that concerns the HIM manager is: a. Admissibility b. Disclosures c. Availability d. Data integrity - ✔✔Correct Answer: D Data integrity services ensure the data are not altered as they are stored or transmitted electronically (Johns 2011, 184). 70. Medical record completion compliance is a problem at Community Hospital. The number of incomplete charts often exceeds the standard set by the Joint Commission, risking a type I violation. Previous HIM committee chairpersons tried multiple methods to improve compliance, including suspension of privileges and deactivating the parking garage keycard of any physician in poor standing. To improve compliance, which of the following would be the next step to overcoming noncompliance? a. Discuss the problem with the hospital CEO. b. Call the Joint Commission. c. Contact other hospitals to see what methods they use to ensure compliance. d. Drop the issue because noncompliance is always a problem. - ✔✔Correct Answer: C The HIM manager may compare organizational data with external data from peer groups to determine best practices (Johns 2011, 609). 71. Identify CPT code(s) for the following Medicare patient. A 67-year-old female undergoes a fine needle aspiration of the left breast with ultrasound guidance to place a localization clip during a breast biopsy. a. 10022 b. 10022, 19295-LT c. 10022, 19295-LT, 76942 d. 10022, 76942 - ✔✔Correct Answer: C Fine needle aspiration with image guidance is coded with 10022. Instructional note directs coder to assign 19295 for placement of localization clip during a breast biopsy. Add radiology code 76942 for supervision and interpretation of ultrasound guidance for localization clip guidance. See instructional notes following code 10022 (AMA 2012b, 59). 72. Identify the ICD-9-CM code for diaper rash, elderly patient. a. 690.10 b. 691.0 c. 782.1 d. 705.1 - ✔✔Correct Answer: B Index Rash, diaper. ICD-9-CM classifies dermatitis to categories 690-694. Atopic dermatitis and related conditions are specific to category 691. Fourth-digit subcategories include diaper or napkin rash and other atopic dermatitis and related conditions (Schraffenberger 2012, 292). 73. Identify the ICD-9-CM diagnostic code(s) for the following: threatened abortion with hemorrhage at 15 weeks; home undelivered. a. 640.01, 640.91 b. 640.03 c. 640.83 d. 640.80 - ✔✔Correct Answer: B Index Abortion, threatened 640.0. Refer to the ICD-9-CM Tabular List (640-649) for the correct fifth digit of 3, antepartum condition, not delivered (Schraffenberger 2012, 274-275). 74. To comply with Joint Commission standards, the HIM director wants to ensure that history and physical examinations are documented in the patient's health record no later than 24 hours after admission. Which of the following would be the best way to ensure the completeness of health records? a. Retrospectively review each patient's medical record to make sure history and physicals are present. b. Review each patient's medical record concurrently to make sure history and physicals are present and meet the accreditation standards. c. Establish a process to review medical records immediately on discharge. d. Do a review of records for all patients discharged in the previous 60 days. - ✔✔Correct Answer: B The benefit of concurrent review is that content or authentication issues can be identified at the time of patient care and rectified in a timely manner (Johns 2011, 410). 75. A patient requests copies of her personal health information on CD. When the patient goes home, she finds that she cannot read the CD on her computer. The patient then requests the hospital provide the medical records in paper format. How should the hospital respond? a. Provide the medical records in paper format b. Burn another CD because this is hospital policy c. Provide the patient with both paper and CD copies of the medical record d. Review the CD copies with the patient on a hospital computer - ✔✔Correct Answer: A The covered entity must provide access to the personal health information in the form or format requested when it is readily producible in such form for format. When it is not readily producible in the form or format requested, it must be produced in a readable hard-copy form or such other form or format agreed upon by the covered entity and the individual (Johns 2011, 831). 76. Which payment system was introduced in 1992 and replaced Medicare's customary, prevailing, and reasonable (CPR) payment system? a. Diagnosis-related groups b. Resource-based relative value scale system c. Long-term care drugs d. Resource utilization groups - ✔✔Correct Answer: B The RBRVS system is the federal government's payment system for physicians. It is a system of classifying health services based on the cost of furnishing physicians' services in different settings, the skill and training levels required to perform the services, and the time and risk involved (Casto and Layman 2011, 151). 77. Identify the CPT code(s) for the following patient: A 2-year-old male presented to the hospital to have his gastrostomy tube changed under fluoroscopic guidance. a. 43752 b. 43760 c. 43761, 76000 d. 49450 - ✔✔Correct Answer: D Code 49450 includes replacement of gastrostomy or cecostomy tube, percutaneous, under fluoroscopic guidance including contrast injections(s), image documentation, and report. Therefore, it would not be appropriate to add code 76000 for fluoroscopic guidance, which is already included in the procedure code (AMA 2012b, 258). 78. A family practitioner requests the opinion of a physician specialist in endocrinology who reviews the patient's health record and examines the patient. The physician specialist records findings, impressions, and recommendations in which type of report? a. Consultation b. Medical history c. Physical examination d. Progress notes - ✔✔Correct Answer: A The consultation report documents the clinical opinion of a physician other than the primary or attending physician. The report is based on the consulting physician's examination of the patient and a review of his or her health record (Johns 2011, 78). 79. Which of the following is (are) the correct ICD-9-CM code(s) for thoracoscopic lobectomy of left lung? a. 32.30 b. 32.41 c. 32.49 d. 34.02, 32.41 - ✔✔Correct Answer: B Index Lobectomy, lung, segmental (with resection of adjacent lobes), thoracoscopic. Segmental includes the complete excision of a lobe of the lung (Schraffenberger 2012, 227-228). 80. An individual designated as an inpatient coder may have access to an electronic medical record to code the record. Under what access security mechanism is the coder allowed access to the system? a. Role-based b. User-based c. Context-based d. Situation-based - ✔✔Correct Answer: A Role-based access control (RBAC) is a control system in which access decisions are based on the roles of individual users as part of an organization (Brodnik et al. 2009, 211). 81. Which part of the problem-oriented medical record is used by many facilities that have not adopted the whole problem-oriented format? a. Problem list as an index b. Initial plan c. SOAP form of progress notes d. Database - ✔✔Correct Answer: C The Subjective, Objective, Assessment, Plan (SOAP) notes are part of the problem-oriented medical records (POMR) approach most commonly used by physicians and other healthcare professionals. SOAP notes are intended to improve the quality and continuity of client services by enhancing communication among healthcare professionals (Odom-Wesley et al. 2009, 217). 82. Which of the following software applications would be used to aid in the coding function in a physician ' s office? a. Grouper b. Encoder c. Pricer d. Diagnosis calculator - ✔✔Correct Answer: B An encoder is a computer software program designed to assist coders in assigning appropriate clinical codes and helps ensure accurate reporting of diagnoses and procedures (LaTour and Eichenwald Maki 2010, 318-319). 83. Which of the following fails to meet the CMS classification of a hospital-acquired condition? a. Stage I pressure ulcers b. Falls and trauma c. Catheter-associated infection d. Vascular catheter-associated infection - ✔✔Correct Answer: A Stage I and II pressure ulcers are not considered hospital-acquired conditions but stage III and IV are (Johns 2011, 326). 84. HIM coding professionals and the organizations that employ them have the responsibility to not tolerate behavior that adversely affects data quality. Which of the following is an example of behavior that should not be tolerated? a. Assign codes to an incomplete record with organizational policies in place to ensure codes are reviewed after the records are complete. b. Follow-up on and monitor identified problems. c. Evaluate and trend diagnoses and procedure code selections. d. Report data quality review results to organizational leadership, compliance staff, and the medical staff. - ✔✔Correct Answer: A The coder is not following established policies (Johns 2011, 265-267). 85. Which classification system is in place to reimburse home health agencies? a. MS-DRGs b. RUGs c. HHRGs d. APCs - ✔✔Correct Answer: C Home health resource groups (HHRGs) represent the classification system established for the prospective reimbursement of covered home care services to Medicare beneficiaries during a 60-day episode of care (Johns 2011, 334). 86. Identify CPT code(s) for the following patient. A 35-year-old female undergoes an excision of a 3.0-cm tumor of her forehead. An incision is made through the skin and subcutaneous tissue. The tumor is dissected free of surrounding structures. The wound is closed in layers with interrupted sutures. a. 21012 b. 21012, 12052 c. 21014 d. 21014, 12052 - ✔✔Correct Answer: A CPT code 21012 describes excision of a subcutaneous soft tissue tumor of the face or scalp greater than 2 cm and is appropriately coded when the tumor is removed from the subcutaneous tissue rather than subgaleal or intramuscular. Simple and intermediate closure of the wound is included in the procedure for the excision in the musculoskeletal section of CPT (AMA 2010a, 28-29; AMA 2012b, 88, 94-95). 87. Identify the correct diagnosis ICD-9-CM code(s) for a patient who arrives at the hospital for outpatient laboratory services ordered by the physician to monitor the patient's Coumadin levels. A prothrombin time (PT) is performed to check the patient's long-term use of his anticoagulant treatment. a. V58.83, V58.61 b. V58.83, V58.63 c. V58.61, 790.92 d. V58.61 - ✔✔Correct Answer: A V58.83, Encounter for therapeutic drug monitoring, is the correct code to use when a patient visit is for the sole purpose of undergoing a laboratory test to measure the drug level in the patient's blood or urine or to measure a specific function to assess the effectiveness of the drug. V58.83 may be used alone if the monitoring is for a drug that the patient is on for only a brief period, not long term. However, there is a Use Additional Code note after code V58.83 to remind the coder to use the additional code for any associated long-term drug use with codes V58.61-V58.69 (Schraffenberger 2012, 450-451). 88. Identify the ICD-9-CM procedure code(s) for insertion of dual chamber cardiac pacemaker and atrial and ventricular leads. a. 37,83, 37.73 b. 37.83, 37.71 c. 37.81, 37.73, 37.71 d. 37.83, 37.72 - ✔✔Correct Answer: D ICD-9-CM classifies cardiac pacemakers to code 37.8: Insertion, replacement, removal, and revision of pacemaker device. In coding initial insertion of a permanent pacemaker, two codes are required—one for the pacemaker (37.80-37.83) and one for the lead (37.70-37.74) (Schraffenberger 2012, 204-205). 89. Several key principles require appropriate physician documentation to secure payment from the insurer. Which answer (listed here) fails to impact payment based on physician responsibility? a. The health record should be complete and legible. b. The rationale for ordering diagnostic and other ancillary services should be documented or easily inferred. c. Documenting the charges and services on the itemized bill. d. The patient's progress and response to treatment and any revision in the treatment plan and diagnoses should be documented. - ✔✔Correct Answer: C The documentation of the charges and itemized bill is not the responsibility of the physician (Smith 2012, 7-8). 90. The technology commonly used for automated claims processing (sending bills directly to third-party payers) is: a. Optical character recognition b. Bar coding c. Neural networks d. Electronic data interchange - ✔✔Correct Answer: D EDI allows the transfer (incoming and outgoing) of information directly from one computer to another by using flexible, standard formats (Johns 2011, 348). 91. Two patients were hospitalized with bacterial pneumonia. One patient was hospitalized for three days and the other patient was hospitalized for 30 days. Both cases result in the same DRG with different lengths of stay. Which answer most closely describes how the hospital will be reimbursed? a. The hospital will receive the same DRG for both patients but additional reimbursement will be allowed for the patient who stayed 30 days because the length of stay was greater than the geometric length of stay for this DRG. b. The hospital will receive the same reimbursement for the same DRG regardless of the length of stay. c. The hospital can appeal the payment for the patient who was in the hospital for 30 days because the cost of care was significantly higher than the average length of stay for the DRG payment. d. The hospital will receive a day outlier for the patient who was hospitalized for 30 days. - ✔✔Correct Answer: B The hospital will receive the same reimbursement regardless of the length of stay (Casto and Layman 2011, 12). 92. This is a statement sent by third-party payers to the patient to explain services provided, amounts billed, and payments made by the health plan. a. Coordination of benefits (COB) b. Explanation of benefits (EOB) c. Medicare summary notice (MSN) d. Remittance advice (RA) - ✔✔Correct Answer: B An EOB is a statement sent by a third-party payer to the patient to explain the services provided (Johns 2011, 343). 93. Identify the ICD-9-CM code(s) for infected ingrown nail. a. 703.0 b. 703.8, 681.11 c. 681.11 d. 681.9 - ✔✔Correct Answer: A Index Ingrowing, nail (finger) (toe) (infected) (Schraffenberger 2012, 295). 94. The patient had a total abdominal hysterectomy with bilateral salpingo-oophorectomy. The coder assigned the following codes: 58150, Total abdominal hysterectomy, with/without removal of tubes and ovaries 58700, Salpingectomy, complete or partial, unilateral/bilateral (separate procedure) What error has the coder made by using these codes? a. Maximizing b. Upcoding c. Unbundling d. Optimizing - ✔✔Correct Answer: C Unbundling is the practice of coding services separately that should be coded together as a package because all parts are included within one code and, therefore, one price. Unbundling, done deliberately, could be considered fraud (Kuehn 2012, 347). 95. Bob Smith was admitted to Mercy Hospital on June 21. The physical was completed on June 23. According to Joint Commission standards, which statement applies to this situation? a. The record is not in compliance because the physical examination must be completed within 24 hours of admission. b. The record is not in compliance because the physical examination must be completed within 48 hours of admission. c. The record is in compliance because the physical examination must be completed within 48 hours. d. The record is in compliance because the physical examination was completed within 72 hours of admission. - ✔✔Correct Answer: A According to the Joint Commission, the physical examination must be completed within 24 hours of admission (Odom-Wesley et al. 2009, 353). 96. The Medicare Modernization Act of 2003 (MMA) launched a Medicare payment and recovery demonstration project that would later develop into recovery audit contractors (RACs) serving as a means to ensure correct payments under Medicare. During the demonstration program, the contractors were able to identify _____ of dollars in improper payments. a. Hundreds b. Thousands c. Millions d. Billions - ✔✔Correct Answer: D The RAC demonstration uncovered $1.03 billion of improper payments, of which 96% were overpayments and 4% were underpayments (Casto and Layman 2011, 39). 97. The documentation of each patient encounter should include the following to secure payment from the insurer except: a. The reason for the encounter and the patient's relevant history, physical exam, and prior diagnostic test results b. A patient assessment, clinical impression, or diagnosis c. A plan of care d. The identity of the patient's nearest relative and emergency contact number - ✔✔Correct Answer: D The identity of the patient's nearest relative and an emergency contact number are not relative to securing payment from the insurer. The encounter should include the date of the encounter and the identity of the observer (Smith 2012, 8). 98. Identify the ICD-9-CM diagnosis code(s) for neonatal tooth eruption. a. 525.0 b. 520.6, 525.0 c. 520.9 d. 520.6 - ✔✔Correct Answer: D Index Eruption, teeth/tooth, neonatal. Some main terms are followed by a list of indented subterms (modifiers) that affect the selection of an appropriate code for a given diagnosis. The subterms form individual line entries arranged in alphabetical order and printed in a regular type beginning with a lowercase letter. Subterms are indented on standard indention to the right under the main term. More specific subterms are further indented after the preceding subterm (Schraffenberger 2012, 12). 99. Identify the correct ICD-9-CM procedure code(s) for replacement of an old dual pacemaker with a new dual pacemaker. a. 37.87 b. 37.85 c. 37.87, 37.89 d. 37.85, 37.89 - ✔✔Correct Answer: A When a pacemaker is replaced with another pacemaker, only the replaced pacemaker is coded (37.85- 37.87). Removal of the old pacemaker is not coded (Schraffenberger 2012, 204-205). 100. The release of information function requires the HIM professional to have knowledge of: a. Clinical coding principles b. Database development c. Federal and state confidentiality laws d. Human resource management - ✔✔Correct Answer: C Because federal regulations such as HIPAA and state laws govern the release of health record information, HIM department personnel must know what information needs to be included on the authorization for it to be considered valid (Johns 2011, 443). 1. Data security policies and procedures should be reviewed at least: a. Semi-annually b. Annually c. Every two years d. Quarterly - ✔✔Correct Answer: B All data security policies and procedures should be reviewed and evaluated at least every year to make sure they are up-to-date and still relevant to the organization (Johns 2011, 995). 2. Identify the correct ICD-9-CM diagnosis code(s) for a patient with near-syncope event and nausea. a. 780.2 b. 780.2, 787.02 c. 780.2, 787.01 d. 780.4, 787.02 - ✔✔Correct Answer: B Near-syncope and nausea are both signs and symptoms and therefore not integral to the other. Both conditions should be coded (Hazelwood and Venable 2012, 71). 3. The codes in the musculoskeletal section of CPT may be used by: a. Orthopedic surgeons only b. Orthopedic surgeons and emergency department physicians c. Any physician d. Orthopedic surgeons and neurosurgeons - ✔✔Correct Answer: C Any physician may use the codes in any section of CPT (AHIMA 2012a, 587). 4. In an EHR, what is the risk of copying and pasting? a. Reduction in the time required to document b. The system not recording who entered the data c. Quicker overall system response time d. System thinking that the original documenter recorded the note - ✔✔Correct Answer: B The system not recording who entered the data (Johns 2011, 433). 5. Mr. Smith is seen in his primary care physician's office for his annual physical examination. He has a digital rectal examination and is given three small cards to take home and return with fecal samples to screen for colorectal cancer. Assign the appropriate CPT code to report this occult blood sampling. a. 82270 b. 82271 c. 82272 d. 82274 - ✔✔Correct Answer: A CPT code 82270 describes a test for occult blood using feces source for the purpose of neoplasm screening with the use of three cards or single triple card for consecutive collection (AMA 2012b, 417). 6. Identify the punctuation mark that is used to supplement words or explanatory information that may or may not be present in the statement of a diagnosis or procedure in ICD-9-CM coding. The punctuation does not affect the code number assigned to the case. The punctuation is considered a nonessential modifier, and all three volumes of ICD-9-CM use them. a. Parentheses ( ) b. Square brackets [ ] c. Slanted brackets [ ] d. Braces { } - ✔✔Correct Answer: A Parentheses enclose supplementary words or explanatory information that may or may not be present in the statement of a diagnosis or procedure. They do not affect the code number assigned in the case. Terms in parentheses are considered nonessential modifiers, and all three volumes of ICD-9-CM use them. Bronchiectasis (fusiform) (postinfectious) (recurrent) is an example of a diagnosis statement with nonessential modifiers noted with parentheses (Schraffenberger 2012, 26-28). 7. Documentation regarding a patient's marital status; dietary, sleep, and exercise patterns; and use of coffee, tobacco, alcohol, and other drugs may be found in the: a. Physical examination record b. History record c. Operative report d. Radiological report - ✔✔Correct Answer: B A complete medical history documents the patient's current complaints and symptoms and lists his or her past medical, personal, and family history (Johns 2011, 63). 8. If an orthopedic surgeon attempted to reduce a fracture but was unsuccessful in obtaining acceptable alignment, what type of code should be assigned for the procedure? a. A "with manipulation" code b. A "without manipulation" code c. An unlisted procedure code d. An E/M code only - ✔✔Correct Answer: A The "with manipulation" code is used because the fracture was manipulated, even if the manipulation did not result in clinical anatomic alignment. See Musculoskeletal Guidelines, Definitions (AHIMA 2012a, 597). 9. What is the maximum number of diagnosis codes that can appear on the UB-04 paper claim form locator 67 for a hospital inpatient principal and secondary diagnoses? a. 35 b. 25 c. 18 d. 9 - ✔✔Correct Answer: B As of January 1, 2011, CMS allows a total of 25 ICD-9-CM diagnosis codes (one principal and 24 additional diagnoses) for 837 Institutional claims filing (Schraffenberger 2012, 66). 10. What type of standard establishes methods for creating unique designations for individual patients, healthcare professionals, healthcare provider organizations, and healthcare vendors and suppliers? a. Vocabulary standard b. Identifier standard c. Structure and content standard d. Security standard - ✔✔Correct Answer: B Identifier standards establish methods for assigning a unique identifier to individual patients, healthcare professionals, healthcare provider organizations, and healthcare vendors and suppliers (Odom-Wesley et al. 2009, 311). 11. Identify the correct ICD-9-CM diagnosis code for a patient with anterolateral wall myocardial infarction, initial episode. a. 410.11 b. 410.01 c. 410.02 d. 410.12 - ✔✔Correct Answer: B Index Infarction, myocardium, anterolateral (wall) with fifth digit for initial episode (Schraffenberger 2012, 26-28). 12. A patient has two health insurance policies: Medicare and a Medicare supplement. Which of the following statements is true? a. The patient receives any monies paid by the insurance companies over and above the charges. b. Monies paid to the healthcare provider cannot exceed charges. c. The decision on which company is primary is based on remittance advice. d. The patient should not have a Medicare supplement. - ✔✔Correct Answer: B The monies collected from third-party payers cannot be greater than the amount of the provider's charges (Johns 2011, 343). 13. Identify the ICD-9-CM diagnosis code(s) for uncontrolled type II diabetes mellitus; mild malnutrition. a. 250.02 b. 250.01, 263.1 c. 250.02, 263.1 d. 250.01, 263.0 - ✔✔Correct Answer: C Diabetes (without complication) with fifth digit of 2 = type II, uncontrolled. 263.1 Malnutrition, mild, not stated as related to diabetes (Schraffenberger 2012, 122-124). 14. Identify the correct sequence and ICD-9-CM diagnosis code(s) for a patient with a scar on the right hand secondary to a laceration sustained two years ago. a. 709.2 b. 906.1 c. 709.2, 906.1 d. 906.1, 709.2 - ✔✔Correct Answer: C The residual condition or nature of the late effect is sequenced first, followed by the cause of the late effect (Hazelwood and Venable 2012, 60-61). 15. Which of the following is the concept responsible for limiting disclosure of private matters including the responsibility to use, disclose, or release such information only with the knowledge and consent of the individual? a. Privacy b. Bioethics c. Security d. Confidentiality - ✔✔Correct Answer: D Confidentiality is the responsibility for limiting disclosure (Johns 2011, 755). 16. Tissue transplanted from one individual to another of the same species but different genotype is called a(n): a. Autograft b. Xenograft c. Allograft or allogeneic graft d. Heterograft - ✔✔Correct Answer: C Tissue transplanted from one individual to another of the same species but different genotype is called an allograft or allogeneic graft (AHIMA 2012a, 592-593). 17. Where would a coder who needed to locate the histology of a tissue sample most likely find this information? a. Pathology report b. Progress notes c. Nurse's notes d. Operative report - ✔✔Correct Answer: A Histology refers to the tissue type of a lesion. The histology of tissue is determined by a pathologist and documented in the pathology report (Johns 2011, 77). 18. A patient with known COPD and hypertension under treatment was admitted to the hospital with symptoms of a lower abdominal pain. He undergoes a laparoscopic appendectomy and develops a fever. The patient was subsequently discharged from the hospital with a principal diagnosis of acute appendicitis and secondary diagnoses of postoperative infection, COPD, and hypertension. Which of the following diagnoses should not be tagged as POA? a. Postoperative infection b. Appendicitis c. COPD d. Hypertension - ✔✔**Correct Answer: A Present on admission is defined as present at the time the order for inpatient admission occurs (CMS 2011c, 97). 19. A hospital needs to know how much Medicare paid on a claim so they can bill the secondary insurance. What should the hospital refer to? a. Explanation of benefits b. Medicare Summary Notice c. Remittance advice d. Coordination of benefits - ✔✔Correct Answer: C Remittance advice (RA) is sent to the provider to explain payments made by third-party payers (Johns 2011, 346). 20. Which of the following would be the best technique to ensure that registration clerks consistently use the correct notation for assigning admission date in an electronic health record (EHR)? a. Make admission date a required field b. Provide an input mask for entering data in the field c. Make admission date a numeric field d. Provide sufficient space for input of data - ✔✔Correct Answer: B When several people enter data in an EHR, you can define how users must enter data in specific fields to help maintain consistency. For example, an input mask for a form means that users can only enter the date in a specified format (MacDonald 2007, chapter 4). 21. Which of the following threatens the "need-to-know" principle? a. Backdating progress notes b. Blanket authorization c. HIPAA regulations d. Surgical consent - ✔✔Correct Answer: B A blanket authorization is a common ethical problem when misused. Patients often sign a blanket authorization, which authorizes the release of information from that point forward, without understanding the implications. The problem is the patient is not aware of what information is being accessed (Johns 2011, 778-779). 22. A fee schedule is: a. Developed by third-party payers and includes a list of healthcare services, procedures, and charges associated with each b. Developed by providers and includes a list of healthcare services provided to a patient c. Developed by third-party payers and includes a list of healthcare services provided to a patient d. Developed by providers and lists charge codes - ✔✔Correct Answer: A A fee schedule is a list of healthcare services and procedures and charges associated with each (Johns 2011, 350). 23. Identify the correct ICD-9-CM diagnosis code for a male patient with stress urinary incontinence. a. 625.6 b. 788.30 c. 788.32 d. 788.39 - ✔✔Correct Answer: C Index Incontinence, stress, male, NEC 788.32. Category 788.3x indicates incontinence of urine with the fifth digit specific to different types such as urge, stress, mixed, and others (Hazelwood and Venable 2012, 73). 24. Identify the correct ICD-9-CM diagnosis codes and sequence for a patient who was admitted to the outpatient chemotherapy floor for acute lymphocytic leukemia. During the procedure, the patient developed severe nausea with vomiting and was treated with medications. a. 204.00, 787.01, V58.11 b. V58.11, 204.00, 787.01 c. V58.11, 204.00 d. 204.22, 787.01 - ✔✔Correct Answer: B When a patient is admitted for the purpose of radiotherapy, chemotherapy, or immunotherapy and develops a complication, such as uncontrolled nausea and vomiting or dehydration, the principal diagnosis is the admission for radiotherapy (V58.0), the admission for the antineoplastic chemotherapy (V58.11), or the admission for the antineoplastic immunotherapy (V58.12). Additional codes would include the cancer and the complication(s) (Hazelwood and Venable 2012, 103). 25. Category II codes cover all but one of the following topics. Which is not addressed by Category II codes? a. Patient management b. New technology c. Therapeutic, preventative, or other interventions d. Patient safety - ✔✔Correct Answer: B New technology is addressed by the Category III codes (AHIMA 2012a, 584). 26. The hospital is revising its policy on medical record documentation. Currently, all entries in the medical record must be legible, complete, dated, and signed. The committee chairperson wants to add that, in addition, all entries must have the time noted. However, another clinician suggests that adding the time of notation is difficult and rarely may be correct since personal watches and hospital clocks may not be coordinated. Another committee member agrees and says only electronic documentation needs a time stamp. Given this discussion, which of the following might the HIM director suggest? a. Suggest that only hospital clock time be noted in clinical documentation b. Suggest that only electronic documentation have time noted c. Inform the committee that according to the Medicare Conditions of Participation, all documentation must be authenticated and dated d. Inform the committee that according to the Medicare Conditions of Participation, only medication orders must include date and time - ✔✔Correct Answer: C All entries must be legible and complete, and must be authenticated and dated promptly by the person (identified by name and discipline) who is responsible for ordering, providing, or evaluating the service furnished (42 CFR 482.24). 27. How are amendments handled in an EHR? a. Automatically appended to the original note; no additional signature is required. b. Amendments must be entered by the same person as the original note. c. Amendments cannot be entered after 24 hours of the event's occurrence. d. The amendment must have a separate signature, date, and time. - ✔✔Correct Answer: D The addendum must have a separate signature, date, and time from the original entry (Johns 2011, 437). 28. What penalties can be enforced against a person or entity that willfully and knowingly violates the HIPAA Privacy Rule with the intent to sell, transfer, or use PHI for commercial advantage, personal gain, or malicious harm? a. A fine of not more than $10,000 only b. A fine of not more than $10,000, not more than 1 year in jail, or both c. A fine of not more than $5,000 only d. A fine of not more than $250,000, not more than 10 years in jail, or both - ✔✔Correct Answer: D When a person or entity willfully and knowingly violates the HIPAA Privacy Rule, a fine of not more than $250,000, not more than 10 years in jail, or both may be imposed (LaTour and Eichenwald Maki 2010, 292). 29. Which of the following reports includes names of the surgeon and assistants, date, duration and description of the procedure, and any specimens removed? a. Operative report b. Anesthesia report c. Pathology report d. Laboratory report - ✔✔Correct Answer: A An operative report describes the surgical procedures performed on the patient (Johns 2011, 73). 30. A provision of the law that established the resource-based relative value scale (RBRVS) stipulates that refinements to relative value units (RVUs) must maintain: a. Moderate rate increases b. Market basket increases c. Budget neutrality d. Sustainable growth rate - ✔✔Correct Answer: C Budget neutrality must be maintained annually when the RVUs are adjusted (Casto and Layman 2011, 156). 31. CPT was developed and is maintained by: a. CMS b. AMA c. Cooperating parties d. WHO - ✔✔Correct Answer: B The AMA developed and maintains CPT. CMS developed and maintains HCPCS Level II codes (AHIMA 2012a, 586). 32. Identify the correct ICD-9-CM diagnosis code for a patient with an elevated prostate specific antigen (PSA) test result. a. 796.4 b. 790.6 c. 792.9 d. 790.93 - ✔✔Correct Answer: D Review Tabular List: Findings, abnormal, without diagnosis, prostate specific antigen (PSA), 790.93, or Elevation, prostate specific antigen (PSA), 790.93 (Hazelwood and Venable 2012, 69). 33. Identify the correct ICD-9-CM diagnosis code(s) and proper sequencing for urinary tract infection due to E. coli. a. 599.0 b. 599.0, 041.49 c. 041.49 d. 041.49, 599.0 - ✔✔Correct Answer: B Connecting words or connecting terms are subterms that indicate a relationship between the main term and an associated condition or etiology in the Alphabetic Index. The connecting term "due to" connects the organism E. coli to the urinary tract infection. The instructional note "Use additional code" is found in the Tabular List of ICD-9-CM. This notation indicates that use of an additional code may provide a more complete picture of the diagnosis or procedure. The additional code should always be assigned if the health record provides supportive documentation. Infection, urinary (tract) Tabular List— use additional code to identify organism. Infection, Escherichia coli (Schraffenberger 2012, 22-23, 79). 34. What is it called when a Medicare hospital inpatient admission results in exceptionally high costs when compared to other cases in the same DRG? a. Rate increase b. Charge outlier c. Cost outlier d. Day outlier - ✔✔Correct Answer: C To qualify for a cost outlier, a hospital's charges for a case (adjusted to cost) must exceed the payment rate for the MS-DRG by a specific threshold amount determined by CMS for each fiscal year (Johns 2011, 374). 35. Health insurance for spouses, children, or both is known as: a. Dependent (family) coverage b. Individual (single) coverage c. Group coverage d. Inclusive coverage - ✔✔Correct Answer: A Health insurance for spouses, children, or both is known as dependent (family) coverage (Casto and Layman 2011, 5). 36. In a routine health record quantitative analysis review, it was found that a physician dictated a discharge summary on 1/26/20XX. The patient, however, was discharged two days later. In this case, what would be the best course of action? a. Request that the physician dictate another discharge summary. b. Have the record analyst note the date discrepancy. c. Request the physician dictate an addendum to the discharge summary. d. File the record as complete because the discharge summary includes all of the pertinent patient information. - ✔✔Correct Answer: C An addendum may be included in the medical record to update or supplement documentation that has been recorded (AHIMA 2008b, 83-88). 37. Observation E/M codes (99218-99220) are used in physician billing when: a. A patient is admitted and discharged on the same date. b. A patient is admitted for routine nursing care following surgery. c. A patient does not meet admission criteria. d. A patient is referred to a designated observation status. - ✔✔Correct Answer: D See instructional notes preceding code 99217. In order to report these codes, the admission order must designate observation status. Whether the patient meets admission criteria or is admitted following surgery does not affect the observation code selection. If the patient is admitted and discharged on the same date, codes 99234-99236 are appropriate (AMA 2012b, 13). 38. When coding a selective catheterization in CPT, how are codes assigned? a. One code for each vessel entered b. One code for the point of entry vessel c. One code for the final vessel entered d. One code for the vessel of entry and one for the final vessel, with intervening vessels not coded - ✔✔Correct Answer: C The only vessel coded is the final vessel entered. See instructional note preceding code 36000. Intermediate steps along the way are not reported (AHIMA 2012a, 604). 39. The Privacy Rule establishes that a patient has the right of access to inspect and obtain a copy of his or her PHI: a. For as long as it is maintained b. For six years c. Forever d. For 12 months - ✔✔Correct Answer: A An individual's right extends for as long as the record is maintained (Johns 2011, 827). 40. Assignment of benefits is a contract between a physician and Medicare in which the physician agrees to bill Medicare directly for covered services and to bill the beneficiary only for ________ , and to accept the Medicare payment as payment in full. a. Coinsurance or deductible b. Deductible only c. Coinsurance only d. Balance of charges - ✔✔Correct Answer: A When a physician accepts assignment of benefits, the physician can only collect any applicable deductible and/or coinsurance from the patient (Casto and Layman 2011, 156). 41. The purpose of a physician query is to: a. Identify the MS-DRG b. Identify the principal diagnosis c. Improve documentation for patient care and proper reimbursement d. Increase reimbursement as form of optimization - ✔✔Correct Answer: C Improve documentation to support services billed (Johns 2011, 348). 42. Identify the correct ICD-9-CM diagnosis code(s) for a patient with seizures; epilepsy ruled out. a. 780.39 b. 345.9 c. 780.39, 345.9 d. 345.90 - ✔✔Correct Answer: A Code signs and symptoms when a condition is ruled out, which means the condition has been proven not to exist. The code for seizures (780.39) is assigned when a more specific diagnosis cannot be made even after all the facts bearing on the case have been investigated (Hazelwood and Venable 2012, 68- 73). 43. The Privacy Rule applies to: a. All covered entities involved with transmitting or performing any electronic transactions specified in the act b. Healthcare providers only c. Only healthcare providers that receive Medicare reimbursement d. Only entities funded by the federal government - ✔✔Correct Answer: A The Privacy Rule is applicable to all covered entities involved, either directly or indirectly, with transmitting or performing any electronic transactions specified in the act (Johns 2011, 823). 44. Which answer FAILS to provide a requirement for assignment of the MS-DRG? a. Diagnoses and procedures (principal and secondary) b. Attending and consulting physicians c. Presence of major or other complications and comorbidities (MCC or CC) d. Discharge disposition or status - ✔✔**Correct Answer: B Attending and consulting physicians have no bearing on the assignment of the MS-DRG and payment to the hospital (Schraffenberger 2012, 471-473). 45. Who is responsible for writing and signing discharge summaries and discharge instructions? a. Attending physician b. Head nurse c. Primary physician d. Admitting nurse - ✔✔Correct Answer: A The physician principally responsible for the patient's hospital care writes and signs the discharge summary (Odom-Wesley et al. 2009, 200). 46. Under the HIPAA privacy standard, which of the following types of protected health information (PHI) must be specifically identified in an authorization? a. History and physical reports b. Operative reports c. Consultation reports d. Psychotherapy notes - ✔✔Correct Answer: D The distinction of psychotherapy notes is important due to HIPAA requirements that these notes may not be released unless specifically specified in an authorization (Odom-Wesley et al. 2009, 440). 47. In hospitals, automated systems for registering patients and tracking their encounters are commonly known as ______ systems. a. MIS b. CDS c. ADT d. ABC - ✔✔Correct Answer: C Automated systems for registering patients and tracking their encounters are commonly known as admission-discharge-transfer (ADT) systems (Johns 2011, 947). 48. Community Hospital implemented a clinical document improvement (CDI) program six months ago. The goal of the program was to improve clinical documentation to support quality of care, data quality, and HIM coding accuracy. Which of the following would be best to ensure that everyone understands the importance of this program? a. Request that the CEO write a memorandum to all hospital staff. b. Give the chairperson of the CDI committee authority to fire employees who don't improve their clinical documentation. c. Include ancillary clinical and medical staff in the process. d. Request a letter from the Joint Commission. - ✔✔Correct Answer: C Staff participation in the process of developing and implementing a program will contribute to the staff understanding of the importance of the program (Russo 2010, chapter 6). 49. Statements that define the performance expectations and/or structures or processes that must be in place are: a. Rules b. Policies c. Guidelines d. Standards - ✔✔Correct Answer: D Standards are fixed rules that must be followed, which is different from a guideline that provides general direction (Johns 2011, 416). 50. What was the goal of the MS-DRG system? a. To improve Medicare's capability to recognize severity of illness in its inpatient hospital payments. The new system is projected to increase payments to hospitals for services provided to sicker patients and decrease payments for treating less severely ill patients. b. To improve Medicare's capability to recognize poor quality of care and pay hospitals on an incentive grid that allows hospitals to be paid by performance. c. To improve Medicare's capability to recognize groups of data by patient populations, which will further allow Medicare to adjust the hospitals wage indexes based on the data. This adjustment will be a system to pay hospitals fairly across all geographic locations. d. To improve Medicare's capability to recognize practice patterns among hospitals that are inappropriately optimizing payments by keeping patients in the hospital longer than the median length of stay. - ✔✔**Correct Answer: A For fiscal year 2008, Medicare adopted a severity-adjusted diagnosis-related groups system called Medicare Severity-DRGs (MS-DRGs). This was the most drastic revision to the DRG system in 24 years. The goal of the new MS-DRG system was to significantly improve Medicare's ability to recognize severity of illness in its inpatient hospital payments. The new system is projected to increase payments to hospitals for services provided to the sicker patients and decrease payments for treating less severely ill patients (Schraffenberger 2012, 471-473). 51. A hospital is planning on allowing coding professionals to work at home. The hospital is in the process of identifying strategies to minimize the security risks associated with this practice. Which of the following would be best to ensure that data breaches are minimized when the home computer is unattended? a. User name and password b. Automatic session terminations c. Cable locks d. Encryption - ✔✔Correct Answer: B Automatic session termination will help to control access to the computer when unattended by automatically ending the session when not in use, preventing unauthorized access (HHS 2006a). 52. What healthcare organizations collect UHDDS data? a. All outpatient settings including physician clinics and ambulatory surgical centers b. All outpatient settings including cancer centers, independent testing facilities, and nursing homes c. All non-outpatient settings including acute-care, short-term care, long-term care, and psychiatric hospitals; home health agencies; rehabilitation facilities; and nursing homes d. All inpatient settings and outpatient settings with a focus on ambulatory surgical centers - ✔✔Correct Answer: C The Uniform Hospital Discharge Data Set was promulgated by the US Department of Health, Education, and Welfare in 1974 as a minimum, common core of data on individual acute-care, short-term hospital discharges in Medicare and Medicaid programs. It sought to improve the uniformity and comparability of hospital discharge data. In 1985, the data was expanded to include all nonoutpatient settings (Schraffenberger 2012, 63-65). 53. What should a hospital do when a state law requires more stringent privacy protection than the federal HIPAA privacy standard? a. Ignore the state law and follow the HIPAA standard b. Follow the state law and ignore the HIPAA standard c. Comply with both the state law and the HIPAA standard d. Ignore both the state law and the HIPAA standard and follow relevant accreditation standards - ✔✔Correct Answer: C When a state law is more stringent than a federal law, hospitals must comply with both (Odom-Wesley et al. 2009, 68). 54. What should be done when the HIM department's error or accuracy rate is deemed unacceptable? a. A corrective action should be taken. b. The problem should be treated as an isolated incident. c. The formula for determining the rate may need to be adjusted. d. Re-audit the problem area. - ✔✔Correct Answer: A Corrective action should be taken when error or accuracy rates are deemed to be at an unacceptable rate (Johns 2011, 417). 55. Identify the correct ICD-9-CM diagnosis code(s) for a patient with sepsis due to Staphylococcus aureus septicemia. a. 038.11, 995.91 b. 995.91, 038.11 c. 038.11 d. 038.11, 995.92 - ✔✔Correct Answer: A 038.11, Septicemia, Staphylococcus aureus, and 995.91, Sepsis. The "Code first" note following code 995.91 directs the coder to assign the code for the underlying infection first (Schraffenberger 2012, 80- 81). 56. An HIM professional's ethical obligations: a. Apply regardless of employment site b. Are limited to the employer c. Apply to only the patient d. Are limited to the employer and patient - ✔✔Correct Answer: A HIM ethical obligations apply regardless of employment site (Johns 2011, 754). 57. Which of the following provides organizations with the ability to access data from multiple databases and to combine the results into a single questions-and-reporting interface? a. Client-server computer b. Data warehouse c. Local area network d. Internet - ✔✔Correct Answer: B A data warehouse is a special type of database that consolidates and stores data from various databases (Johns 2011, 909). 58. During a review of documentation practices, the HIM director finds that nurses are routinely using the copy-and-paste function of the hospital's new EHR system for documenting nursing notes. In some cases, nurses are copying and pasting the objective data from the lab system and intake-output records as well as the patient's subjective complaints and symptoms originally documented by another practitioner. Which of the following should the HIM director do to ensure the nurses are following acceptable documentation practices? a. Inform the nurses that "copy and paste" is not acceptable and to stop this practice immediately. b. Determine how many nurses are involved in this practice. c. Institute an in-service training session on documentation practices. d. Develop policies and procedures related to cutting, copying, and pasting documentation in the EHR system. - ✔✔Correct Answer: D In order to thoughtfully and appropriately manage copy functionality, organizations must have sound documentation integrity policies within their organization. HIM professionals should lead their organizations in developing copy policies and procedures that address operational processes, utilization of copy functionality, documentation guidelines, responsibility, and auditing and reporting (AHIMA 2012b, 9-10, 18-21). Documentation policies are used to define the acceptable practices that should be followed by all applicable staff to ensure consistency and continuity and clarity in documentation (AHIMA 2005). 59. Identify the correct ICD-9-CM diagnosis code(s) for a patient with pneumonia and persistent cough. a. 786.2, 490 b. 486, 786.2 c. 486 d. 481 - ✔✔Correct Answer: C Pneumonia, unspecified, is assigned 486 in the Alphabetic Index. Cough is integral to pneumonia and should not be coded separately (Hazelwood and Venable 2012, 68-73). 60. Per CPT guidelines, a separate procedure is: a. Coded when it is performed as part of another, larger procedure b. Considered to be an integral part of another, larger service c. Never coded under any circumstance d. Both a and b - ✔✔Correct Answer: B Because a separate procedure is considered a part of, and integral to, another, larger procedure, it is not coded when performed as part of the more extensive procedure. See Surgery Guidelines. It may, however, be coded when it is not performed as part of another, larger service; therefore, answer "c" is not correct (AHIMA 2012a, 586). 61. The ______ uses expert or artificial intelligence software to automatically assign code numbers. a. Functional EHR b. NHIN c. NLP encoding system d. Grouper - ✔✔Correct Answer: C Natural-language processing (NLP) uses artificial intelligence software to allow digital text from online documents stored in the organization's information system to be read directly by the software, which then suggests codes to match the documentation (Johns 2011, 170). 62. Identify the acute-care record report where the following information would be found: Gross Description: Received fresh designated left lacrimal gland is a single, unoriented, irregular, tan-pink portion of soft tissue measuring 0.8 × 0.6 × 0.1 cm, which is submitted entirely intact in one cassette. a. Medical history b. Medical laboratory report c. Pathology report d. Physical examination - ✔✔Correct Answer: C A pathology report usually includes descriptions of the tissue from a gross or macroscopic level and representative cells at the microscopic level along with interpretive findings (Johns 2011, 77). 63. If a provider believes a service may be denied by Medicare because it could be considered unnecessary, the provider must notify the patient before the treatment begins by using a(n): a. Advance beneficiary notice (ABN) b. Advance notice of coverage (ANC) c. Notice of payment (NOP) d. Consent for payment (CFP) - ✔✔Correct Answer: A An advance beneficiary notice (ABN) must be given to the patient to sign prior to treatment if any indication presents that may cause the service to be denied by Medicare (Johns 2011, 350). 64. Identify the acute-care record report where the following information would be found: The patient is a well-developed, obese male who does not appear to be in any distress, but has considerable problem with mobility. He has difficulty rising up from a chair, and he uses a cane to ambulate. VITAL SIGNS: His blood pressure today is 158/86, pulse is 80 per minute, weight is 204 pounds (which is 13 pounds below what he weighed in April). He has no pallor. He has rather pronounced shaking of his arms, which he claims is not new. NECK: Showed no jugular venous distension. HEART: Very irregular. LUNGS: Clear. EXTREMITIES: Edema of both legs. a. Discharge summary b. Medical history c. Medical laboratory report d. Physical examination - ✔✔Correct Answer: D The physical examination report represents the attending physician's assessment of the patient's current health status (Johns 2011, 63). 65. Reimbursement for healthcare services is dependent on patients having a(n): a. Attending physician b. Insurance benefit c. Explanation of benefits d. Qualified provider - ✔✔Correct Answer: B Generally, reimbursement for healthcare services is dependent on patients having health insurance (Casto and Layman 2011, 3). 66. Which of the following ethical principles is being followed when an HIT professional ensures that patient information is only released to those who have a legal right to access it? a. Autonomy b. Beneficence c. Justice d. Nonmaleficence - ✔✔Correct Answer: B Beneficence means promoting good (Johns 2011, 1113). 67. In coding arterial catheterizations, when the tip of the catheter is manipulated from the insertion into the aorta and then out into another artery, this is called: a. Selective catheterization b. Nonselective catheterization c. Manipulative catheterization d. Radical catheterization - ✔✔Correct Answer: A If the tip of the catheter is manipulated, it is a selective catheterization. In the case of a nonselective catheterization, the tip of the catheter remains in either the aorta or the artery that was originally entered (AHIMA 2012a, 604). 68. Which of the following is a threat to data security? a. Encryption b. People c. Red flags d. Access controls - ✔✔Correct Answer: B Threats to data security caused by people can be classified as threats from insiders who make unintentional mistakes, threats from insiders who abuse their access privileges to information, threats from insiders who access information or computer systems for spite or profit, threats from insiders who attempt to access information or steal physical resources, and from vengeful employees or outsiders who mount attacks on the organization's information systems (Johns 2011, 987). 69. The provider or supplier is prohibited from holding the patient responsible for charges in excess of the Medicare fee schedule. This is called: a. Accept assignment b. Balance billing c. Charge capture d. Inducement - ✔✔Correct Answer: B Balance billing means the patient cannot be held responsible for charges in excess of the Medicare fee schedule (Johns 2011, 350). 70. What type of organization works under contract with the CMS to conduct Medicare and Medicaid certification surveys for hospitals? a. Accreditation organizations b. Certification organizations c. State licensure agencies d. Conditions of participation agencies - ✔✔Correct Answer: C State licensure agencies have regulations that are modeled after the Medicare Conditions of Participation and Joint Commission standards. States conduct annual surveys to determine the hospital's continued compliance with licensure standards (Odom-Wesley et al. 2009, 287). 71. An HIT using her password can access and change data in the hospital's master patient index. A billing clerk, using his password, cannot perform the same function. Limiting the class of information and functions that can be performed by these two employees is managed by: a. Network controls b. Audit trails c. Administrative controls d. Access controls - ✔✔Correct Answer: D Access control means being able to identify which employees should have access to what data (Johns 2011, 992). 72. Identify the correct ICD-9-CM diagnosis codes for metastatic carcinoma of the colon to the lung. a. 153.9, 162.9 b. 197.0, 153.9 c. 153.9, 197.0 d. 153.9, 239.1 - ✔✔Correct Answer: C The terms metastatic to and direct extension to are used for classifying secondary malignant neoplasms in ICD-9-CM. For example, cancer described as "metastatic to a specific site" is interpreted as a secondary neoplasm of that site. The colon (153.9) is the primary site, and the lung (197.0) is the secondary site (Hazelwood and Venable 2012, 109). 73. Which organization developed the first hospital standardization program? a. Joint Commission b. American Osteopathic Association c. American College of Surgeons d. American Association of Medical Colleges - ✔✔Correct Answer: C The American College of Surgeons started its Hospital Standardization Program in 1918 (Johns 2011, 679). 74. Identify the correct ICD-9-CM diagnosis code(s) for a patient with nausea, vomiting, and gastroenteritis. a. 558.9 b. 787.01, 558.9 c. 787.02, 787.03, 558.9 d. 787.01, 558.41 - ✔✔Correct Answer: A Conditions that are integral to the disease process should not be assigned as additional codes. The nausea and vomiting are integral to the disease, gastroenteritis (Hazelwood and Venable 2012, 68). 75. Which of the following is NOT an accepted accrediting body for behavioral healthcare organizations? a. American Psychological Association b. Joint Commission c. Commission on Accreditation of Rehabilitation Facilities d. National Committee for Quality Assurance - ✔✔Correct Answer: A The Joint Commission, Commission on Accreditation of Rehabilitation Facilities, and the National Committee for Quality Assurance are all acceptable accrediting bodies for behavioral healthcare settings (Odom-Wesley et al. 2009, 447). 76. Identify the correct ICD-9-CM diagnosis code(s) for a patient with right lower quadrant abdominal pain with nausea, vomiting, and diarrhea. a. 789.03 b. 789.03, 787.02, 787.03, 787.91 c. 789.03, 787.91 d. 789.03, 787.01, 787.91 - ✔✔Correct Answer: D Abdominal pain includes fifth digits to identify the specific parts of the abdomen affected. Nausea and vomiting is a category common to stomach upset. The fifth digits provide specificity. Nausea and vomiting are coded together with a combination code when both exist. Diarrhea usually is a symptom of some other disorder or of a more severe disease, in which case it should not be coded separately. It is often accompanied by vomiting and various other symptoms that should be coded when present. Because, in this case, a distinct disease is not available, all the symptoms should be coded (Hazelwood and Venable 2012, 73). 77. Identify the correct CPT procedure code for incision and drainage of infected shoulder bursa. a. 10060 b. 10140 c. 23030 d. 23031 - ✔✔Correct Answer: D Index Incision and drainage, shoulder, bursa, resulting in code 23031 (AHIMA 2012a, 598). 78. How does Medicare or other third-party payers determine whether the patient has medical necessity for the tests, procedures, or treatment billed on a claim form? a. By requesting the medical record for each service provided b. By reviewing all the diagnosis codes assigned to explain the reasons the services were provided c. By reviewing all physician orders d. By reviewing the discharge summary and history and physical for the patient over the last year - ✔✔Correct Answer: B Diagnosis codes are often the primary reason for a service to be considered covered or denied by the insurance company. Local and national policies include diagnosis codes that are used in software edits to automatically deny or approve processed claims. Denied services can be appealed and the record can be submitted to support medical necessity if the service fails the automated review (Schraffenberger 2012, 476). 79. A(n) ______ is computer software that assists in determining coding accuracy and reliability. a. Encoder b. Interface c. Diagnosis related group d. Record locator service - ✔✔Correct Answer: A An encoder is computer software that helps the coding professional assign codes (Johns 2011, 269). 80. Which of the following situations would be identified by the NCCI edits? a. Determining the MS-DRG b. Billing for two services that are prohibited from being billed on the same day c. Whether data submitted electronically were successfully submitted d. Receiving the remittance advice - ✔✔Correct Answer: B Billing for two services that are prohibited from being billed on the same day. (Johns 2011, 347). 81. Identify the correct ICD-9-CM diagnosis code(s) for a patient with an abnormal glucose tolerance test. a. 790.29 b. 790.21 c. 790.21, 790.29 d. 790.22 - ✔✔Correct Answer: D The Index may mislead the coder to a nonspecific code. In this example, when the coder references "Abnormal" and subheading "glucose," the coder is directed to code 790.29. The coder should always reference the Tabular List to verify the code. During verification, the coder will see the selection for code 790.22, which accurately describes the specific abnormal finding of glucose tolerance test (Hazelwood and Venable 2012, 74). 82. A coding analyst consistently enters the wrong code for patient gender in the electronic billing system. What security measures should be in place to minimize this security breach? a. Access controls b. Audit trail c. Edit checks d. Password controls - ✔✔Correct Answer: C Edit checks help ensure data integrity by allowing only reasonable and predetermined values to be entered into the computer (Johns 2011, 509). 83. Mohs micrographic surgery involves the surgeon acting as: a. Both plastic surgeon and general surgeon b. Both surgeon and pathologist c. Both plastic surgeon and dermatologist d. Both dermatologist and pathologist - ✔✔Correct Answer: B See definitions preceding code 17311 (Mohs micrographic technique) in CPT Professional Edition (AMA 2012b, 79). 84. Which of the following is the concept of the right of an individual to be left alone? a. Privacy b. Bioethics c. Security d. Confidentiality - ✔✔Correct Answer: A Privacy is the right of an individual to be left alone (Johns 2011, 755). 85. Which of the following would not be found in a medical history? a. Chief complaint b. Vital signs c. Present illness d. Review of systems - ✔✔Correct Answer: B Medical history documents the patient's current complaints and symptoms and lists the patient's past medical, personal, and family history. The physical examination report represents the attending physician's assessment of the patient's current health status (Johns 2011, 63). 86. When correcting erroneous information in a health record, which of the following is NOT appropriate? a. Print "error" above the entry b. Enter the correction in chronological sequence c. Add the reason for the change d. Use black pen to obliterate the entry - ✔✔Correct Answer: D In a paper-based health record environment, corrections to health record entries are corrected by drawing a single line through the original entry, writing "error" above the entry, and then the practitioner signs, dates, and times the correction (Johns 2011, 413). 87. What is the maximum number of procedure codes that can appear on a UB-04 institutional claim form via electronic transmission? a. 6 b. 9 c. 15 d. 25 - ✔✔Correct Answer: D Effective January 1, 2011, CMS allows a total of 25 ICD-9-CM procedure codes for 837 Institutional claims filing (Schraffenberger 2012, 66). 88. What are possible "add-on" payments that a hospital could receive in addition to the basic Medicare DRG payment? a. Additional payments may be made for locum tenens, increased emergency room services, stays over the average length of stay, and cost outlier cases. b. Additional payments may be made to critical access hospitals, for higher-than-normal volumes, unexpected hospital emergencies, and cost outlier cases. c. Additional payments may be made for increased emergency room services, critical access hospitals, increased labor costs, and cost outlier cases. d. Additional payments may be made to disproportionate share hospitals for indirect medical education, new technologies, and cost outlier cases. - ✔✔Correct Answer: D Medicare provides for additional payment for other factors related to a particular hospital's business. If the hospital treats a high percentage of low-income patients, it receives a percentage add-on payment applied to the MS-DRG adjusted base payment rate. This add-on payment, known as the disproportionate share hospital (DSH) adjustment, provides for a percentage increase in Medicare payments to hospitals that qualify under either of two statutory formulas designed to identify hospitals that serve these areas. Hospitals that have approved teaching hospitals also receive a percentage addon payment for each Medicare discharged paid under IPPS, known as the indirect medical education (IME) adjustment. The percentage varies, depending on the ratio of residents to beds. Additional payments are made for new technologies or medical services that have been approved for special addon payments. Finally, the costs incurred by a hospital for a Medicare beneficiary are evaluated to determine whether the hospital is eligible for an additional payment as an outlier case. This additional payment is designed to protect the hospital from large financial losses due to unusually expensive cases (Schraffenberger 2012, 471-473). 89. What is the name of the national program to detect and correct improper payments in the Medicare Fee-for-Service (FFS) program? a. Medicare administrative contractors (MACs) b. Recovery audit contractors (RACs) c. Comprehensive error rate testing (CERT) d. Fiscal intermediaries (FIs) - ✔✔Correct Answer: B Congress directed HHS to conduct a three-year demonstration project using RACs to detect and correct improper payments in the Medicare traditional fee-for-service program. Congress further required HHS to make the RAC program permanent and nationwide by January 1, 2010 (Schraffenberger 2012, 475). 90. Identify the correct ICD-9-CM diagnosis code(s) and sequence for a patient with disseminated candidiasis secondary to AIDS-like syndrome. a. 042, 112.5, V01.79 b. 112.5, 042 c. 042, 112.5, V08 d. 042, 112.5 - ✔✔Correct Answer: D Only confirmed cases of HIV infection/illness are reported whether inpatient or outpatient. 042, Human immunodeficiency virus [HIV] disease. Patients with HIV-related illness should be coded to category 042, which includes AIDS, AIDS-like syndrome, AIDS-related complex, and symptomatic HIV infection (Hazelwood and Venable 2012, 89-90). 91. What is the name of the organization that develops the billing form that hospitals are required to use? a. American Academy of Billing Forms (AABF) b. National Uniform Billing Committee (NUBC) c. National Uniform Claims Committee (NUCC) d. American Billing and Claims Academy (ABCA) - ✔✔Correct Answer: B The NUBC was established with the goal of developing an acceptable, uniform bill that would consolidate the numerous billing forms hospitals were required to use (Schraffenberger 2012, 65). 92. What is the basic formula for calculating each MS-DRG hospital payment? a. Hospital payment = DRG relative weight × hospital base rate b. Hospital payment = DRG relative weight × hospital base rate − 1 c. Hospital payment = DRG relative weight / hospital base rate + 1 d. Hospital payment = DRG relative weight / hospital base rate - ✔✔Correct Answer: A For any given patient in a MS-DRG, the hospital knows, in advance, the amount of reimbursement it will receive from Medicare. It is the responsibility of the hospital to ensure that its resource use is in line with the payment (Schraffenberger 2012, 471-473). 93. Documentation in the history of use of drugs, alcohol, and/or tobacco is considered part of the: a. Past medical history b. Social history c. Systems review d. History of present illness - ✔✔Correct Answer: B Documentation of history of use of drugs, alcohol, and/or tobacco is considered part of the social history. The review of systems is a part of the history of present illness. See E/M Services Guidelines, instructions for selecting a level of E/M service, in the CPT manual (AMA 2011a, 4-7). 94. Identify the ICD-9-CM diagnosis code(s) for neutropenic fever. a. 288.00 b. 288.00, 780.60 c. 288.01 d. 288.00, 780.61 - ✔✔Correct Answer: D 288.00, Fever, neutropenic. Instructional note states to use additional code for any associated fever (780.61) (Schraffenberger 2012, 137-139). 95. During an audit of health records, the HIM director finds that transcribed reports are being changed by the author up to a week after initial transcription. The director is concerned that changes occurring this long after transcription jeopardize the legal principle that documentation must occur near the time of the event. To remedy this situation, the HIM director should recommend which of the following? a. Immediately stop the practice of changing transcribed reports. b. Develop a facility policy that defines the acceptable period of time allowed for a transcribed document to remain in draft form. c. Conduct a verification audit. d. Alert hospital legal counsel of the practice. - ✔✔Correct Answer: B Documentation policies are used to define the acceptable practices that should be followed by all applicable staff to ensure consistency, continuity, and clarity in documentation (AHIMA 2005). 96. What is a chargemaster? a. Cost-sharing in which the policy or certificate holder pays a preestablished percentage of eligible expenses after the deductible has been met b. A plan that converts the organization's goals and objectives into targets for revenue and spending c. A financial management form that contains information about the organization's charges for the healthcare services it provides to patients d. Charged amounts that are billed as costs by an organization to the current year's activities of operation - ✔✔Correct Answer: C A chargemaster is a financial management form that contains information about the organization's charges for the healthcare services it provides to patients. Answer "a" is coinsurance. Answer "b" is budget. Answer "d" is expense (Johns 2011, 1116). 97. Identify the correct sequence and ICD-9-CM diagnosis code(s) for a patient with dysphasia secondary to old cerebrovascular accident sustained one year ago. a. 787.20, 438.12 b. 784.59, 438.12 c. 438.12 d. 787.20, 438.89 - ✔✔Correct Answer: C The residual condition or nature of the late effect is sequenced first, followed by the cause of the late effect. Late effect exceptions occur when the late effect code has been expanded at the fourth- and fifth-digit level to include the manifestations. In this case, only one code is necessary to describe both the residual condition and cause of the late effect (Hazelwood and Venable 2012, 62). 98. HIPAA regulations: a. Never preempt state statutes b. Always preempt state statutes c. Preempt less-strict state statutes where they exist d. Preempt stricter state statutes where they exist - ✔✔**Correct Answer: C HIPAA regulations preempt less strict state statutes where they exist (Johns 2011, 820). 99. An employee in the physical therapy department arrives early every morning to snoop through the clinical information system for potential information about neighbors and friends. What security mechanisms should be implemented to prevent this security breach? a. Audit controls b. Information access controls c. Facility access controls d. Workstation security - ✔✔Correct Answer: B An EHR can be viewed by multiple users and from multiple locations at any time, and organizations must have in place appropriate security access control measures to ensure the safety of the data (Johns 2011, 435). 100. The ______ is a type of coding that is a natural outgrowth of the EHR. a. Automated codebook b. Computer-assisted coding c. Logic based encoder d. Decision support database - ✔✔Correct Answer: B Computer-assisted coding is defined as the use of computer software that automatically generates a set of medical codes for review, validation, and use based on the documentation from the various providers of healthcare (AHIMA 2010b, 62; LaTour and Eichenwald Maki 2010, 400). 1. The patient, a 47-year-old male with adenoma of the prostate, is being treated in the outpatient surgery suite. The urologist inserts an endoscope in the penile urethra and dilates the structure to allow instrument passage. After endoscope placement, a radiofrequency stylet is inserted, and the diseased prostate is excised with radiant energy. Bleeding is controlled with electrocoagulation. Following instrument removal, a catheter is inserted and left in place. Which of the following code sets will be reported for this service? a. 600.20, 53852 b. 600.20, 52601 c. 600.00, 53852 d. 222.2, 53850 - ✔✔Correct Answer: A When thermotherapy is used code 53852 is reported. Code 52601 is reported for electrosurgical resection; 53850 is reported for radiofrequency. Adenoma of the prostate is reported with 600.20 (AHIMA 2012a, 697). 2. The HIPAA Privacy Rule requirement that covered entities must limit use, access, and disclosure of PHI to the least amount necessary to accomplish the intended purpose. What concept is this an example of? a. Minimum necessary b. Notice of Privacy Practice c. Consent d. Authorization - ✔✔Correct Answer: A The Privacy Rule introduced the standard of minimum necessary, a "need to know" filter that is applied to limit access to a patient's protected health information (PHI) and to limit the amount of PHI used, disclosed, and requested (Brodnik et al. 2009, 176). 3. An infant is born by cesarean section at 27 weeks' gestation. The baby weights 945 g. The baby's lungs are immature, and the baby develops respiratory distress syndrome, requiring a 25-day hospital stay in the NICU. Discharge diagnosis: Extreme immaturity, with 27-week gestation, with respiratory distress syndrome, delivered by cesarean section. Which of the following diagnosis ICD-9-CM codes would be correct? a. V30.01, 765.03, 765.24 b. 765.03, 769 c. V30.01, 765.03, 765.24, 769 d. V30.01, 769 - ✔✔Correct Answer: C The codes for prematurity 765.03 and code 765.24 for weeks of gestation meet reporting guidelines as additional diagnoses. A birth code of V30.01 is reported as the principal diagnosis (CMS 2010c, Section I, C, 15b; AHA 2006, 190). See instructional note under 765.0x to "Use Additional Code" for weeks of gestation. A code is also needed for the respiratory distress syndrome, 769 (AHIMA 2012a, 676) 4. Which of the following statements about Category III CPT codes is false? a. They are temporary codes. b. They are updated more frequently than the rest of the CPT codes. c. They are intended to allow for the coding of new technologies, services, and procedures. d. They are tracking codes that can be used for performance measurement. - ✔✔Correct Answer: D Category II CPT codes are used for performance measurement (AMA 2012b, 535). 5. Which statement fails to be true for Medicare coverage? a. Medicare pays for healthcare services provided to Social Security beneficiaries 65 years old and older b. Medicare pays for healthcare services provided to Social Security beneficiaries for new moms 65 years and younger and their newborn babies c. Medicare pays for healthcare services provided to Social Security beneficiaries for people under 65 years old with certain disabilities d. Medicare pays for healthcare services provided to Social Security beneficiaries for people of all ages with end-stage renal disease - ✔✔Correct Answer: B Medicare does not cover moms and newborn babies unless the mother has a disability. Moms and newborn babies can be covered under the Medicaid program if they meet specific income guidelines (Johns 2011, 293, 301). 6. A health information technician has been asked to design a problem list for an electronic health record (EHR). Which of the following data elements should be included on the problem list? a. Problem number, problem description, date problem entered b. Problem number, problem name, date of consent for treatment c. Patient identifying information, problem number, examination results d. Problem name, date of onset, physical exam - ✔✔Correct Answer: A The problem list describes any significant current and past illnesses and conditions as well as the procedures the patient has undergone (Johns 2011, 94). 7. Identify the CPT procedure code(s) for an automated CBC with automated differential. a. 85027 b. 85025 c. 85041 d. 85007, 85025 - ✔✔Correct Answer: B Index Blood Cell Count, hemogram, added indices, resulting in code range 85025-85027. The codes for reporting CBCs (complete blood counts) are very specific and should be carefully reviewed. The appropriate code for a CBC with automated white blood cell differential is 85025 (AHIMA 2012a, 628). 8. Who is responsible for ensuring the quality of health record documentation? a. Board of directors b. Administrator c. Provider d. Health information management professional - ✔✔Correct Answer: C The provider is responsible for ensuring the quality of the documentation of the healthcare record (Brodnik et al. 2009, 128). 9. Identify the CPT procedure code(s) and correct modifier for a thyroid stimulating hormone (TSH) when medical necessity is not met and the patient signs a required waiver of liability signifying the patient will be responsible for payment if the test is not covered by Medicare. Another name for waiver of liability is Advance Beneficiary Notice (ABN). a. 84443-GA b. 80418-GA c. 84443-GY d. 80418-GY - ✔✔Correct Answer: A Index Thyroid simulating hormone, 80418, 80438-80440, 84443. Code 84443 is the correct code for a TSH while the rest of the codes are panels including several tests. Modifier -GA is listed in the front cover of the CPT Professional Edition and signifies the patient was given a notice of non-coverage also known as waiver of liability or ABN (AMA 2012b, 427; CMS 2010d; CMS 2010e). 10. A Medicare Advantage Plan (like an HMO or PPO) is a health coverage option under what part of Medicare? a. Part A b. Part B c. Part C d. Part E - ✔✔Correct Answer: C Medicare Part C combines Medicare Part A and Medicare Part B coverage and is operated by private insurance companies that are approved by and under contract with Medicare to form Medicare Advantage Plans (Johns 2011, 293). 11. The use of computer software that automatically generates a set of medical codes for review, validation, and use based on clinical documentation provided by healthcare practitioners is the definition of: a. Natural language processing b. Voice recognition c. Computer-assisted coding d. Electronic health record - ✔✔Correct Answer: C Computer-assisted coding utilizes computer software to generate codes from the data provided (Sayles and Trawick 2010, 360). 12. In developing a coding compliance program, which of the following would not be ordinarily included as participants in coding compliance education? a. Current coding personnel b. Medical staff c. Newly hired coding personnel d. Nursing staff - ✔✔Correct Answer: D The cornerstone of accurate coding is physician documentation. Ensuring the accuracy of coded data is a shared responsibility between coding professional and physicians (Johns 2011, 357). 13. A hospital allows the use of the copy functionality in its EHR system for documentation purposes. The hospital has established explicit policies that define when the copy function may be used. Which of the following would be the best approach for conducting a retrospective analysis to determine if hospital copy policies are being followed? a. Randomly audit EHR documentation for patients readmitted within 30 days b. Survey practitioners to determine if they are following hospital policy c. Institute an in-service program for all hospital personnel d. Observe the documentation practices of all clinical personnel - ✔✔Correct Answer: A Hospitals must randomly audit EHR documentation to ensure compliance with hospital policy. Readmissions within 30 days serve as a good patient sample for the copy function in the EHR (AHIMA 2012b) 14. The patient presented through the ED with severe abdominal pain, amenorrhea. Serum human chorionic gonadotropin (hCG) was lower than normal. There were also endometrial and uterine changes. Patient diagnosed with tubal pregnancy. A unilateral salpingectomy with removal of tubal pregnancy was performed. Which of the following is the correct code assignment? a. 633.80, 66.62 b. 633.10, 66.62 c. 633.10, 66.4 d. 633.10, 66.02 - ✔✔Correct Answer: B The ectopic pregnancy was documented as tubal. The salpingectomy was "with removal of tubal pregnancy." The procedure performed was a salpingectomy, not a salpingostomy (AHIMA 2012a, 679). 15. Identify the CPT procedure code(s) for ultrasound, pregnant uterus, fetal and maternal evaluation, second trimester, single gestation. a. 76700 b. 76805 c. 76801 d. 76813 - ✔✔Correct Answer: B Index Ultrasound, pregnant uterus, resulting in code range 76801-76817. Review of the available codes indicates that code 76805 is the appropriate code (AHIMA 2012a, 625). 16. The computer abstracting system in a facility has an edit that does not allow coders to assign obstetrical codes to male patients. This edit is called a(n): a. Self-correcting control b. Feedback control c. Presence of a virus d. Audit trail - ✔✔Correct Answer: A Preventive controls are front-end processes that guide work in such a way that input and process variations are minimized. Simple things such as standard operating procedures, edits on data entered into computer-based systems, and training processes are ways to reduce the potential for error by using preventive controls (also called self-correcting controls) (LaTour and Eichenwald Maki 2010, 696). 17. CCI edit files contain code pairs, called mutually exclusive edits, which prevent payment for: a. Services that cannot reasonably be billed together b. Services that are components of a more comprehensive procedure c. Unnecessary procedures d. Comprehensive procedures - ✔✔Correct Answer: A The mutually exclusive edit applies to improbable or impossible combinations of codes (Johns 2011, 347). 18. A patient is scheduled for a colonoscopy, but due to sudden drop in blood pressure, the procedure is canceled just as the scope is introduced into the rectum. Because of moderately severe mental retardation, the patient is given a general anesthetic prior to the procedure. How should this procedure be coded by the hospital? a. Assign the code for a colonoscopy with modifier -74. b. Assign the code for a colonoscopy with modifier -52. c. Assign an anesthesia code only. d. Do not assign a code because no procedure was performed. - ✔✔Correct Answer: A Per CPT coding guidelines, when a planned procedure is terminated prior to completion for cause, the intended procedure is coded with a modifier. See instructions for use of modifiers in Appendix A. When a procedure is terminated after the induction of anesthesia, modifier -74 is appended to the intended procedure. See Medicare billing requirements for specific rules for canceled endoscopy procedures (AMA 2012b; AHIMA 2012a, 635). 19. In determining the data collection requirements for Medicare patients in a long-term care facility, the health information technician would consult standards from: a. CARF b. CMS c. The Joint Commission d. NCQA - ✔✔Correct Answer: B The CMS has Conditions of Participation that apply to healthcare organizations that participate in the Medicare program (Johns 2011, 98). 20. A quantitative review of the health record for missing reports and signatures that occurs when the patient is in the hospital is referred to as a: a. Prospective review b. Retrospective review c. Concurrent review d. Peer review - ✔✔Correct Answer: C Quantitative analysis can occur concurrently while the patient is in the hospital (Johns 2011, 410). 21. Where would information on treatment given on a particular encounter be found in the health record? a. Problem list b. Physician's orders c. Progress notes d. Physical examination - ✔✔Correct Answer: C Progress notes are chronological statements about the patient's response to treatment during his or her stay at the facility (Kuehn 2011, 10). 22. Identify the CPT procedure code(s) for a SPECT bone scan. a. 78710 b. 78803 c. 78607 d. 78320 - ✔✔Correct Answer: D Index Bone, nuclear medicine, SPECT, resulting in code 78320. The acronym SPECT stand for single photon emission computed tomography and is a more sophisticated form of CT scanning. Unlike basic xray CT scanning, SPECT involves injected radionuclides and is considered a form of nuclear medicine. It is being supplanted to some extent now by PET (positron emission tomography) scanning, which is capable of better resolution and sensitivity (AHIMA 2012a, 625). 23. What kind of care offers extensive psychiatric treatment on an outpatient basis with the expectation that the patient's level of functioning will improve so that hospitalization can be avoided? a. Acute hospitalization b. Partial hospitalization c. Outpatient day care d. Short-term care nursing - ✔✔Correct Answer: B Comprehensive outpatient rehabilitation facility services and mental healthcare provided as part of a partial hospitalization psychiatric program when a physician certifies that inpatient treatment would be required without the partial hospitalization services (Johns 2011, 296). 24. Select the appropriate CPT code(s) to report a therapeutic subcutaneous injection of rabies immune globulin performed under direct physician supervision. a. 96372 b. 90471 c. 90375, 96372 d. 90375, 90473 - ✔✔Correct Answer: C In order to appropriately report administration of vaccines, both the product administered and the method of administration must be reported. An instructional note listed before CPT code 90476 states: "(For immune globulins, see codes 90281-90399, 96365-96368, 96372-96375 for administration of immune globulins)" (AMA 2012b, 459-460). 25. Dr. Jones has signed a statement that all of her dictated reports should be automatically considered approved and signed unless she makes corrections within 72 hours of dictating. This is called: a. Autoauthentication b. Electronic signature c. Automatic record completion d. Chart tracking - ✔✔Correct Answer: A Autoauthentication is a policy that allows the physician or provider to state in advance that dictated and transcribed reports should automatically be considered approved and signed when the physician does not make corrections within a certain period of time. Another variation of autoauthentication is that physicians authorize the HIM department to send a weekly list of documents needing signatures. The list is then signed and returned to the HIM department (LaTour and Eichenwald Maki 2010, 213). 26. Which of the following personnel should be authorized, per hospital policy, to take a physician's verbal order for the administration of medication? a. Unit secretary working on the unit where the patient is located b. Nurse working on the unit where the patient is located c. Health information director d. Admissions registrars - ✔✔Correct Answer: B Because of the risks associated with miscommunication, verbal orders are discouraged. When a verbal order is necessary, a clinician should sign, give his or her credential (for example, RN, PT, or LPN), and record the date and time the order was received. Verbal orders for medication are usually required to be given to, and to be accepted only by, nursing or pharmacy personnel (Brodnik et al. 2009, 131). 27. Which of the following statements does not pertain to electronic health records (EHRs)? a. EHR technologies and systems must not intrude on the patient and provider relationship. b. EHRs are filed in paper folders. c. In the United States, a national health information infrastructure is being designed to support EHRs. d. Clinicians may use computer keyboards when documenting in the EHR. - ✔✔Correct Answer: B EHRs store information in electronic format rather than paper-based media (Johns 2011, 135). 28. Identify the CPT procedure code(s) for extracorporeal sound wave lithotripsy of large kidney stone. a. 50590 b. 52353 c. 43265 d. 28890 - ✔✔Correct Answer: A Index Lithotripsy, kidney, resulting in code 50590 or 52353. Review of the available codes indicates that code 50590 is correct because there is no mention of cystourethroscopy (AHIMA 2012a, 609). 29. Which is a feature of managed care? a. Control and reduce the costs of care b. Monitor the activity of physician supervision c. Provide incentive for prospective payment d. Allow the patient to choose several primary physicians - ✔✔Correct Answer: A The purpose of managed care is to control or reduce the costs of healthcare for which the third-party payer must reimburse the providers and to ensure continuing quality of care (Casto and Layman 2011, 9). 30. Which of the following is often cited as a reason to implement an electronic health record (EHR)? a. Improve patient safety b. High cost of EHR c. Staff time required to implement the EHR d. Simplicity of implementation changes to workflow - ✔✔Correct Answer: A The primary benefits of EHRs are quality and patient safety (Johns 2011, 173). 31. Who may sign an authorization for use and disclosure when the patient is a minor? a. The minor's parent or legal guardian b. The patient c. The physician d. The social worker - ✔✔Correct Answer: A As a general rule, minors are legally incompetent and unable to make decisions regarding the use and disclosure of their own healthcare information. This authority belongs to the minor's parent(s) or legal guardians(s) unless an exception applies (Brodnik et al. 2009, 243). 32. A 75-year-old male patient was admitted for an acute exacerbation of chronic systolic congestive heart failure and severe mitral regurgitation and aortic stenosis. What would be the correct code assignment for this case? a. 428.23, 396.2 b. 428.23, 428.0, 396.2 c. 428.0, 394.1, 424.1 d. 391.8, 396.2 - ✔✔Correct Answer: B Code 428.23, 428.0 and 396.2 2 would be the correct codes with 428.23 serving as the principal diagnosis. Code 428.23 is described as systolic heart failure in acute and chronic conditions. The code for mitral valve insufficiency and aortic valve stenosis is a combination code of 396.2. Code 428.0 is not an inherent component of diastolic or systolic heart failure and must be coded separately (AHIMA 2012a, 651). 33. An audit of a hospital's electronic health system shows that diagnostic codes are not being reported at the correct level of detail. This indicates a problem with: a. Data granularity b. Data consistency c. Data comprehensiveness d. Data relevancy - ✔✔Correct Answer: A Data granularity requires that the attributes and values of data be defined at the correct level of detail for the intended use of the data (Johns 2011, 48). 34. A physician takes the medical records of a group of HIV-positive patients out of the hospital to complete research tasks at home. The physician mistakenly leaves the records in a restaurant, where they are read by a newspaper reporter who publishes an article that identifies the patients. The physician can be sued for: a. Slander b. Willful infliction of mental distress c. Libel d. Invasion of privacy - ✔✔Correct Answer: D A person's right to privacy is implied in the US Constitution and is "the right to be left alone—the right to be free from unwarranted publicity and exposure to public view, as well as the right to live one's life without having one's name, picture, or private affairs made public against one's will." The right to privacy is also the right to control personal information (Pozgar 2009, 36; LaTour and Eichenwald Maki 2010, 277). 35. The inpatient prospective payment system for MS-DRG assignment begins with the: a. Principal diagnosis b. Primary diagnosis c. Secondary diagnosis d. Surgical procedure - ✔✔Correct Answer: A To determine the appropriate MS-DRG, a claim for a healthcare encounter is first classified into one of the 25 major diagnostic categories, or MDCs. The principal diagnosis determines the MDC assignment (Johns 2011, 322-323). 36. This program was initiated by the Balanced Budget Act of 1997 and allows states to expand existing insurance programs to cover children up to age 19. a. Children's State Medicare Program (CSMP) b. State Children's Health Insurance Program (SCHIP)) c. Children's State Healthcare Alliance (CSHA) d. Children's Aid to Healthcare (CAH) - ✔✔Correct Answer: B The State Children's Health Insurance Program (SCHIP) (Title XXI of the Social Security Act) provides additional federal funds to states so that Medicaid eligibility can be expanded to include a greater number of children (Johns 2011, 304). 37. An HIM professional who releases health information that he or she knows will result in genetic discrimination is violating the ethical principle of: a. Autonomy b. Beneficence c. Justice d. Nonmaleficence - ✔✔Correct Answer: D Nonmaleficence means to do no harm. With regard to the patient and the healthcare team, the HIM professional is obligated to protect health, medical, genetic, social, personal, financial, and adoption information: Clinical information (genetic risk factors) must be protected as well as behavioral information. It is important to protect genetic and social information so that patients will not be vulnerable to the risks of discrimination (LaTour and Eichenwald Maki 2010, 311-312). 38. The present on admission (POA) indicator is a requirement for a. Inpatient Medicare claims submitted by acute care hospitals b. Inpatient Medicare and Medicaid claims submitted by hospitals c. Medicare claims submitted by all entities d. Inpatient skilled nursing facility Medicare claims - ✔✔Correct Answer: A The POA indicator applies to diagnosis codes for claims involving inpatient admission to acute-care hospitals or other facilities, as required by law or regulation for public health reporting (Schraffenberger 2012, 58; CMS 2011c, 97-102; Johns 2011, 325). 39. Which of the following software applications would be used to aid in the coding function in a physician's office? a. Grouper b. Encoder c. Pricer d. Diagnosis calculator - ✔✔Correct Answer: B An encoder is a computer software program designed to assist coders assign appropriate clinical codes. An encoder helps ensure accurate reporting of diagnoses and procedures (LaTour and Eichenwald Maki 2010, 318-319). 40. An infusion that lasts less than 15 minutes would be reported with a(n): a. Intravenous infusion code b. Intravenous piggyback code c. Intravenous or intra-arterial push code d. Intravenous hydration code - ✔✔Correct Answer: C An infusion that lasts less than 15 minutes should be reported with an IV push code per the CPT coding guidelines of the CPT Professional Edition based on the instructional notes preceding the hydration notes (AMA 2012b, 518; AHIMA 2012a, 630). 41. Patient data collection requirements vary according to healthcare setting. One would expect a data element would be collected in the MDS, but would not be collected in the UHDDS. a. Personal identification b. Cognitive patterns c. Procedures and dates d. Principal diagnosis - ✔✔Correct Answer: B According to UHDDS requirements, answers a, c, and d represent items collected about inpatients. Only answer b represents a data item collected more typically in long-term care settings and required in the MDS (Johns 2011, 98). 42. Identify the CPT procedure code(s) and correct modifiers for a basic metabolic panel with ionized calcium and an additional test for carbon dioxide on the same day. Keep in mind that a carbon dioxide test is part of the basic metabolic panel. a. 80047 b. 80047, 82374 c. 80047, 82374-91 d. 80048, 82374-91 - ✔✔Correct Answer: C Index Organ or disease-oriented panel, metabolic, basic to reference codes 80047 and 80048. Code 80047 includes ionized calcium for the correct panel code. Index Carbon Dioxide, blood or urine to arrive at code 82374. Add modifier -91 to the carbon dioxide code to signify the test was performed twice on the same day and is separately reportable. Modifier -91 represents a repeat clinical diagnostic laboratory test, and should be used for a repeat test even when the first test is part of a panel represented by a panel code (AMA 2012b, 402). 43. Identify the CPT procedure code(s) for laparoscopic Nissen fundoplication. a. 43279 b. 43280 c. 43327 d. 43289 - ✔✔Correct Answer: B Index Fundoplasty, esophagogastric, laparoscopic, resulting in code 43280. Indexing the main term Nissen operation results in this cross-reference: see fundoplasty, esophagogastric, laparoscopic, results in code 43280 (AHIMA 2012a, 608). 44. For coding and billing professionals, being compliant means to perform one's job functions according to the laws, regulations and guidelines with integrity as set forth by Medicare and other third-party payers. This is an example of:: a. Ethics b. Skills c. Behaviors d. Education - ✔✔Correct Answer: A, Following the AHIMA Standard of Ethical Coding, sets forth guidelines that all coding and billing professionals understand in ethical decision making (Casto and Layman 2011, 34). 45. Under local anesthesia and ultrasound guidance, a patient underwent radiofrequency ablation of an incompetent greater saphenous vein in the right lower extremity. Assign the appropriate CPT code(s). a. 36475-RT b. 36475-RT, 36000 c. 36478-RT d. 36475-RT, 76942 - ✔✔Correct Answer: A The introduction of the catheter into the vein is included in the procedure code, per the instructional note following code 36476. Code 36478 describes laser ablation of incompetent veins. The scenario for coding specifies radiofrequency ablation, 36475. Per the description of code 36475, the procedure code is inclusive of all imaging guidance and monitoring (AMA 2011b; AHIMA 2012a, 691). 46. Which dimension of data quality is defined as "data that is free of errors?" a. Accuracy b. Granularity c. Precision d. Currency - ✔✔Correct Answer: A Data that are free of errors are accurate. Typographical errors in discharge summaries or misspellings of names are examples of inaccurate data (LaTour and Eichenwald Maki 2010, 119). 47. An intentional representation that an individual knowingly does to be false and knowing that the act could result in some unauthorized benefit to some other person is an example of: a. Abuse b. Ethical behavior c. Fraud d. Exploding Act - ✔✔Correct Answer: C Fraud is an intentional representation that an individual knows to be false or does not believe to be true and knowingly misrepresents the act which could result in an unauthorized benefit (Casto and Layman 2011, 34). 48. Which of the following specialized patient assessment tools must be used by Medicare-certified home care providers? a. Patient Assessment Instrument b. Minimum Data Set for Long-Term Care c. Resident Assessment Protocol d. Outcomes and Assessment Information Set - ✔✔Correct Answer: D Medicare-certified home healthcare use a standardized patient assessment instrument called the OASIS (Johns 2011, 100). 49. When the CCI editor flags that a comprehensive code and a component code are billed together for the same beneficiary on the same date of service, Medicare will pay for: a. The component code but not the comprehensive code b. The comprehensive but not the component code c. The comprehensive and the component codes d. Neither the comprehensive nor the component codes - ✔✔Correct Answer: B The component code is integral to the comprehensive code and should not be billed separately (CMS 2012a). 50. The HIM director is having difficulty with the on-call physicians in the emergency services department completing their health records. Currently, three deficiency notices are sent to the physicians including an initial notice, a second reminder, and a final notification. Which of the following would be the best first step in trying to rectify the current situation? a. Routinely send out a fourth notice b. Post the hospital policy in the emergency department c. Consult with the physician in charge of the on-call doctors for suggestions on how to improve response to the current notices d. Call the Joint Commission - ✔✔Correct Answer: C As part of the decision making process, the HIM director should analyze the problem and develop alternative solutions (Johns 2011, 410-412). 51. Identify the appropriate CPT code(s) for 23 minutes of therapeutic exercise. a. 97110 b. 97110, 97110 c. 97110, 97110, 97110 d. 97110-50 - ✔✔Correct Answer: B Index Physical Medicine/Therapy/Occupational Therapy, procedures, therapeutic exercises, resulting in code 97110. Review of the code indicates that it is reported in 15-minute increments. Thus, a 23- minutes session would be reported with code 97110 twice because a unit of time must be at least 8 minutes at a minimum, which the second unit meets the 8-minute minimum (AHIMA 2012a, 633). 52. The patient was admitted with increasing shortness of breath, weakness, and nonproductive cough. Treatment included oxygen therapy. Final diagnoses listed as acute respiratory insufficiency and acute exacerbation of chronic obstructive pulmonary disease (COPD). Which of the following is the correct ICD-9-CM diagnostic code assignment? a. 491.21 b. 491.21, 518.82 c. 518.81, 491.21 d. 518.82, 491.21 - ✔✔Correct Answer: A Acute respiratory insufficiency is an integral part of COPD and is therefore not coded separately. The patient had acute respiratory insufficiency and not acute respiratory failure (AHIMA 2012a, 682). 53. What kind of care is covered when a patient requires nursing or rehab services occurring within 30 days of a 3-day stay or longer in an acute care hospital setting and is certified as medically necessary? a. Skilled nursing facility care b. Home health care c. Hospice care d. Acute healthcare - ✔✔Correct Answer: A Skilled nursing care (SNF) is covered when a patient requires skilled nursing or rehab services within 30 days of a 3-day or longer acute care hospitalization stay (Johns 2011, 295). 54. One objective of the Balanced Budget Act (BBA) of 1997 was to: a. Improve program integrity for Medicare by educating beneficiaries to report errors noticed on their explanation of benefits (EOBs) to the Department of Health and Human Services (HHS) b. Improve the quality of care to its beneficiaries by increasing availability to healthcare c. Streamline healthcare costs into one type of payment system for Medicare and Medicaid d. Educate hospital providers how to manage quality care with less reimbursement - ✔✔Correct Answer: A The Balanced Budget Act of 1997 was incorporated to improve program integrity for Medicare by educating Medicare beneficiaries to be on the watch for errors in billing of services they didn't receive and any other forms of fraudulent activity (Casto and Layman 2011, 37). 55. MS-DRG may be split into a maximum of _______ payment tiers based on severity as determined by the presence of a major complication/comorbidity, a CC; or no CC. a. Two b. Three c. Four d. Five - ✔✔Correct Answer: B The Medicare IPPS categorizes diagnosis and procedure codes. The diagnosis codes may qualify for a major complication or comorbidity (MCC), or other complication or comorbidity (CC). A diagnosis code may not qualify for either, allowing diagnosis codes to be grouped into three higher or lower DRG groupings (Schraffenberger 2012, 57; Johns 2011, 322-323). 56. A patient was admitted for recurrent dislocation of the shoulder. The operation included debridement of the acromion, subacromial bursectomy, division of the coracoacromial ligament, and an abrasion acromioplasty with Mitek suture placement. Which of the following is the correct code assignment? a. 718.31, 81.82 b. 718.31, 81.82, 83.5 c. 831.00, 81.82, 83.5 d. 831.00, 81.82, 83.5, 80.41 - ✔✔Correct Answer: A Bursectomy and division of ligament are included in acromioplasty. Dislocation is not acute; it is stated as recurrent (AHIMA 2012a, 666). 57. The attending physician is responsible for which of the following types of acute care documentation? a. Consultation report b. Discharge summary c. Laboratory report d. Pathology report - ✔✔Correct Answer: B The results of radiological and pathological procedures require interpretation by specially trained physicians called radiologists and pathologists. These physicians document their findings in written reports. The consultation report documents the clinical opinion of a physician other than the primary or attending physician (Johns 2011, 78). 58. How frequently are Category III CPT codes updated? a. Annually b. Semiannually c. Every two years d. Every four months - ✔✔Correct Answer: B An instructional note has been added to the introductory language under Category III codes in the CPT Professional Edition. "New codes in this section are released semi-annually via the AMA/CPT internet site, to expedite dissemination for reporting. The full set of temporary codes for emerging technology, services, and procedures are published annually in the CPT codebook." (AMA 2012b, 553). 59. What part of Medicare covers Hospital Insurance that helps cover inpatient hospital, skilled nursing, home health, and hospice care? a. Part A b. Part B c. Part C d. Part D - ✔✔Correct Answer: A Medicare Part A Hospital Insurance covers inpatient hospital, skilled nursing, home health, and hospice care (Johns 2011, 293). 60. What is the healthcare expense the patient or insured party is responsible to pay which limited the amount the patient would be responsible for: a. Out-of-pocket expense b. Coinsurance c. Deductible d. Premium - ✔✔Correct Answer: A Out-of-pocket expenses are the healthcare expenses that the insured party is responsible for paying. Insurance companies introduced this high-benefit-level in major medical plans to limit the amount of out-of-pocket expenses to the insured (Johns 2011, 288). 61. Dr. Jones comes into the HIM department and requests the HIM director to pull all of his records from the previous year in which the principal diagnosis of myocardial infarction was indicated. Where would the HIM director begin to pull these records? a. Disease index b. Master patient index c. Operative index d. Physician index - ✔✔Correct Answer: A The disease index is a listing in diagnosis code number order for patients discharged from the facility during a particular time period. Each patient's diagnoses are converted from a verbal description to a numerical code, usually using a coding system such as the ICD-9-CM (LaTour and Eichenwald Maki 2010, 331). 62. What term is used for retrospective cash payments paid by the patient for services rendered by a provider? a. Fee-for-service b. Deductible c. Retrospective d. Prospective - ✔✔Correct Answer: A Patient paid cash for services on a retrospective fee-for-service basis, which meant the patient was expected to pay the healthcare provider after a service was rendered (Johns 2011, 291). 63. Identify the CPT procedure code(s) for whole-body PET scan. a. 78813 b. 78816 c. 78806 d. 78804 - ✔✔Correct Answer: A Index Nuclear medicine, tumor imaging, positron emission tomography, resulting in code range 78811- 78816. Review of the available codes indicates that 78813 is the correct code (AHIMA 2012a, 624). 64. Before healthcare organizations can provide services, they usually must obtain by government entities such as the state in which they are located. a. Accreditation b. Certification c. Licensure d. Permission - ✔✔Correct Answer: C Compliance with state licensing laws is required in order for healthcare organizations to remain in operation (Johns 2011, chapter 61). 65. The National Correct Coding Initiative was developed to control improper coding leading to inappropriate payment for: a. Part A Medicare claims b. Part B Medicare claims c. Medicaid claims d. Medicare and Medicaid claims - ✔✔Correct Answer: B CMS implemented the National Correct Coding Initiative (NCCI) in 1996 to develop correct coding methodologies to improve the appropriate payment of Medicare Part B claims (Johns 2011, 347). 66. What is the name of the formal document prepared by the surgeon at the conclusion of surgery to describe the surgical procedure performed? a. Operative report b. Tissue report c. Pathology report d. Anesthesia record - ✔✔Correct Answer: A The operative report describes the surgical procedures performed on the patient (Johns 2011, 73). 67. The patient is a 45-year-old female who fell while walking her dog. She was walking on the sidewalk in her neighborhood and accidently tripped and subsequently fell. She sustained a comminuted fracture of the shaft of her right tibia confirmed by x-ray done in the emergency room. She also hit her head on a fire hydrant and suffered a slight concussion but no loss of consciousness. The patient was admitted and taken to surgery, where an open reduction with internal fixation was accomplished with good alignment of fracture fragments. Post-op course was uneventful and the patient was discharged with daily physical therapy at home. Which of the following would be coded? a. 823.20, 850.0, E885.9, E019.0, E849.8, E000.8, 79.36 b. 823.10, 850.0, E885.9, E019.0, E849.8, 79.46 c. 823.20, 850.0, E885.9, E849.5, E000.9, 79.36 d. 823.30, 850.0, E885.9, E019.0, E849.8, E000.9, 79.46 - ✔✔Correct Answer: A A comminuted fracture is considered closed unless specified as open or compound per the note in the Index under Fracture. Four "E" codes are necessary to fully describe the circumstances as instructed by the notes in the Tabular list(AHIMA 2012a, 687). 68. Code 87900, infectious agent drug susceptibility phenotype prediction using regularly updated genotypic bioinformatics, is used in the management of patients with what disease? a. Cancer patients on toxic chemotherapy agents b. HIV patients on antiretroviral therapy c. Tuberculosis patients on rifampin therapy d. Organ transplant patients on immunosuppressive therapy - ✔✔Correct Answer: B CPT code 87900 for infectious agent drug susceptibility phenotype prediction using regularly updated genotypic bioinformatics is used in the management of HIV patients on antiretroviral therapy (AMA 2012b, 442). 69. The Medicare Modernization Act (MMA) of 2003 called for CMS to launch a Medicare payment recovery demonstration project. The purpose of the act eventually resulted in the implementation of a group contracted by the government to monitor suspicious and improper activity of Medicare payments including overpayments and underpayments. What is this group? a. Operation Restore Trust b. Payment Error Prevention Program c. Recovery Audit Contractors d. Medicare Administrative Contractors - ✔✔Correct Answer: C Recovery audit contractors (RACs) would become a cost-effective means of ensuring correct payments to providers under Medicare. The RACs were charged with identifying underpayments and overpayments for claims filed under Medicare (Casto and Layman 2011, 39). 70. What is the term used for the record of care in any health-related setting, used by healthcare professionals while providing patient-care services or for administrative, business, or payment purposes? a. Minimum data record b. Legal health record c. Mixed-media health record d. Electronic health record - ✔✔Correct Answer: B The legal health record is the record of care in any health-related setting, used by healthcare professionals while providing patient-care services or for administrative, business, or payment purposes (Odom-Wesley et al. 2009, 24). 71. If a nurse uses the abbreviation CPR to mean cardiopulmonary resuscitation one time and computerbased patient record another time, leading to confusion if the chart were audited would be a concern when applying this dimension of data quality? a. Accuracy b. Granularity c. Precision d. Currency - ✔✔Correct Answer: B Data quality needs to be consistent. A difference in the use of abbreviations provides a good example of how the lack of consistency can lead to problems (LaTour and Eichenwald Maki 2010, 119). 72. Identify the CPT procedure code(s) for a repeat transurethral resection of prostate tissue four years after original procedure. a. 52601 b. 55801 c. 52630 d. 52500 - ✔✔Correct Answer: C Index Excision, prostate regrowth, resulting in code 52630 (AHIMA 2012a, 611). 73. The number of ligatures, sutures, packs, drains, sponges used, and specimens removed would be found in the: a. Anesthesia report b. Progress notes c. Operative report d. Recovery room report - ✔✔Correct Answer: C The operative report describes the surgical procedures performed on the patient (Johns 2011, 73). 74. Which of the following provides a complete description to patients about how PHI is used in a healthcare facility? a. Notice of Privacy Practices b. Authorization c. Consent for treatment d. Minimum necessary - ✔✔Correct Answer: A The NPP explains the patients' rights and the covered entity's legal duties with respect to PHI (Brodnik et al. 2009, 165). 75. Which of the following is a prospective payment system implemented for payment of acute hospital inpatient services? a. APC b. DRG c. OPPS d. RBRVS - ✔✔Correct Answer: C In 1983, CMS implemented a PPS for inpatient hospital care provided to Medicare beneficiaries. The PPS methodology is called diagnosis-related groups (DRGs) (Johns 2011, 319). 76. After a deductible has been paid, insured and insurer share covered losses according to a specified ratio, and the insured pays a(n) _________ amount. a. Out-of-pocket expense b. Coinsurance c. Deductible d. Premium - ✔✔Correct Answer: B Coinsurance refers to the amount the insured pays as a requirement of the insurance policy to share covered losses by insurer and insured (Johns 2011, 288). 77. What kind of provider setting manages patients who are terminally ill and often have a life expectancy less than six months? a. Skilled nursing facility b. Home health c. Hospice d. Acute-care, inpatient - ✔✔Correct Answer: C Hospice care is provided to the terminally ill persons whose life expectancies are certified by their attending physician to be six months or less (Johns 2011, 295). 78. The Joint Commission requires that the medical record delinquency rate quarter averaged from the last four quarterly measurements is not greater than ______ of the average monthly discharge rate. a. 25% b. 50% c. 75% d. 100% - ✔✔Correct Answer: B The Joint Commission will cite a healthcare organization with a requirement for improvement if the total average health record delinquency exceeds 50 percent of the average monthly discharges in any one quarter (Johns 2011, 609). 79. An 8-year-old male hemophiliac is admitted with acute blood loss anemia due to uncontrolled bleeding. He is given clotting factor and six units of whole blood. Which of the following diagnosis and procedure ICD-9-CM codes would be correct? a. 286.0, 99.06, 99.03 b. 285.1, 286.0, 99.06, 99.03 c. 286.0, 285.1, 99.06, 99.03 d. 285.1, 99.06, 99.03 - ✔✔Correct Answer: B The anemia code 285.1 would be coded as the principal diagnosis. In accordance with the UHDDS definition for principal diagnosis, the anemia (not the hemophilia), is the reason for admission and sequenced as the principal diagnosis (CMS 2010c, Section II, 96; AHIMA 2012a, 646). 80. Protection of healthcare information from damage, loss, and unauthorized alteration is also known as: a. Privacy b. Results management c. Security d. Data accuracy - ✔✔Correct Answer: C Security includes physical and electronic protection of the integrity, availability, and confidentiality of computer-based information and the resources used to enter, store, process, and communicate it; and the means to control access and protect information from accidental or intentional disclosure (Johns 2011, 755). 81. What is the amount the insured pays before the insurer assumes liability for any remaining costs of covered services? a. Out-of-pocket expense b. Coinsurance c. Deductible d. Premium - ✔✔Correct Answer: C A deductible is the amount the insured pays before the insurer assumes liability for any remaining costs of covered services (Johns 2011, 288). 82. CPT procedure codes 96360 and 96361 are used to report infusion of: a. Chemotherapeutic agents b. Sequential drugs of the same drug family c. Hormonal antineoplastics d. Prepackaged fluids and/or electrolytes - ✔✔Correct Answer: D Instructional notes for hydration state "Codes 96360-96361 are intended to report a hydration IV infusion to consist of a pre-packaged fluid and electrolytes (eg, normal saline, D5-1/2 normal saline+30mEq KCl/liter), but are not used to report infusion of drugs or other substances." (AMA 2012b, 518-519; AHIMA 2012a, 630). 83. What part of Medicare covers physician services, outpatient care and home healthcare? a. Part A b. Part B c. Part C d. Part D - ✔✔Correct Answer: B Medicare Part B insurance covers physician services, outpatient care, and home healthcare (Johns 2011, 293). 84. In an effort to move toward an episode-of-payment-based payment system, the creation of grouping of APCs were created to allow for multiple services that are typically performed together to be reimbursed by one APC rather than multiple. This methodology is called: a. Composite APC b. Bundling APC c. Discounting APC d. Multiple APC - ✔✔Correct Answer: A CMS added the concept of composite APCs to the hospital outpatient perspective payment system in CY 2008 in an effort to streamline services that are typically performed together and could be grouped into one payment (Casto and Layman 2011, 181). 85. Identify the acute care record report where the following information would be found: "Set up appointment at the Hypertension Center. Hold potassium supplements. Phenergan p.o. 12.5 mg 1-2 tablets p.o. p.r.n. every 6 hrs." a. Medical laboratory report b. Pathology report c. Physical exam d. Physician order - ✔✔Correct Answer: D Physician orders are the instructions the physician gives to the other healthcare professionals (Johns 2011, 63). 86. In which of the following payment systems is the amount of payment determined before the service is delivered? a. Fee-for-service b. Per diem c. Prospective d. Retrospective - ✔✔Correct Answer: C In a prospective payment system (PPS), the exact amount of the payment is determined before the service is delivered (Johns 2011, 315). 87. A ventilator-dependent patient (due to chronic obstructive pulmonary disease emphysema) is admitted to the hospital at 10 a.m. on January 1. He is admitted for dehydration and is placed on the hospital's ventilator upon admission. The patient is discharged January 5 at 1 p.m. What is the appropriate code assignment? a. 492.8, 276.51, 96.72 b. 276.51, 492.8, V46.11, 96.72 c. 276.51, 496, V46.11, 96.71 d. 492.8, 276.51, V46.11, 96.72 - ✔✔Correct Answer: B The dehydration is the reason for admission and should be listed as the principal diagnosis. The mechanical ventilation was for 96 continuous hours or more, code 96.72. The code for the ventilatordependence also should be coded (AHIMA 2012a, 682). 88. Which of the following is true about the Joint Commission's "Do Not Use" abbreviation list? a. Applies to all orders and all medication-related documentation that is handwritten (including free-text computer entry) or on pre-printed forms b. Applies to only medication-related orders c. Applies to all documentation in the health record d. Applies to only preprinted forms - ✔✔Correct Answer: A For accreditation purposes, the official Do Not Use list applies, at a minimum, to all orders and all medication-related documentation that is handwritten (including free-text computer entry) or on preprinted forms. This requirement does not currently apply to preprogrammed health information technology systems (for example, electronic medical records or CPOE systems), but remains under consideration for the future. Organizations contemplating introduction or upgrade of such systems should strive to eliminate the use of dangerous abbreviations, acronyms, symbols, and dose designations from the software (Joint Commission 2009). 89. The Health Insurance Portability and Accountability Act (HIPAA): a. Provides a federal floor for healthcare privacy b. Preempts all state laws c. Applies to anyone who collects health information d. Duplicates Joint Commission standards - ✔✔Correct Answer: A The HIPAA Privacy Rule provides a federal floor on privacy requirements (Johns 2011, 820). 90. Identify the CPT procedure code(s) for partial right-sided thyroid lobectomy with isthmusectomy and subtotal resection of left thyroid. a. 60210 b. 60225 c. 60220 d. 60212 - ✔✔Correct Answer: D Index Thyroidectomy, partial, resulting in code range 60210-60225 or Lobectomy, thyroid gland partial, resulting in code range 60210-60212. Review of the available codes indicates that code 60212 is correct because there is documentation of isthmusectomy and subtotal resection on the opposite (contralateral) side (AHIMA 2012a, 614). 91. The discharge summary must be completed within ________ after discharge for most patients but within ________ for patients transferred to other facilities. Discharge summaries are not always required for patients who were hospitalized for less than ________ hours. a. 30 days/48 hours/24 hours b. 14 days/24 hours/48 hours c. 14 days/48 hours/24 hours d. 30 days/24 hours/48 hours - ✔✔Correct Answer: D The discharge summary must be completed within 30 days after discharge for most patients but within 24 hours for patients transferred to other facilities. Discharge summaries are not always required for patients who are hospitalized for less than 48 hours (Odom-Wesley et al. 2009, 200). 92. Certain services are not covered by Medicare Part A or Part B and may only be covered by private health plans under the Medicare Advantage program. Which service is covered by Medicare Part A or B? a. Dentures and dental care b. Eyeglasses c. X-rays d. Hearing aids - ✔✔Correct Answer: C X-rays are covered under Part A or Part B for hospital and physician services. Dentures, eyeglasses, hearing aids, long-term nursing care and custodial care are not covered by Medicare Part A or B (Johns 2011, 297). 93. A 45-year-old man underwent colon resection for carcinoma of the transverse colon. The physician progress note on postoperative day two states anemia. Hemoglobin and hematocrit levels dropped significantly after surgery, and a blood transfusion was ordered. How is the anemia coded? a. 285.1 b. 998.11 c. 998.11, 285.1 d. Unable to code; the physician must be queried. - ✔✔Correct Answer: D The anemia may be acute blood loss or a complication due to surgery, but it is not stated by the physician. Due to incomplete physician documentation, query the physician (AHA 2004, 4; AHA 2000, 6; AHA 1992, 15-16; AHIMA 2012a, 645). 94. This was passed during the Civil War in order to prohibit contractors of any kind from knowingly filing a false or fraudulent claim, using a false record or statement, or conspiring to defraud the US government. a. American Reinvestment and Recovery Act b. False Claims Act c. False Claims ActBalanced Budget Refinement Act d. Civil Anti-trust Act - ✔✔Correct Answer: B The False Claims Act was passed during the Civil War in an effort to prevent false use of claims and records from defrauding the US government. Today the federal government can rebuke abusers of the Medicare and Medicaid systems (Casto and Layman 2011, 35). 95. What technology creates images of handwritten and printed documents that are then stored in health record databases as electronic files? a. Clinical data repository b. Data exchange standards c. Central processor d. Digital scanner - ✔✔Correct Answer: D Odom-Wesley et al. 2009, 227 96. Patient admitted with cervical lymphadenopathy. Lymph node biopsy confirmed Hodgkin's sarcoma disease. Megavoltage radiotherapy begun. Which of the following is the correct code set? a. 201.21, 40.40, 92.29 b. 201.91, 40.40, 92.29 c. 201.21, 40.11, 92.24 d. 201.91, 40.11, 92.24 - ✔✔Correct Answer: C Hodgkins sarcoma is assigned to code 201.21. Lymph node biopsy is coded to 40.11. Megavoltage radiotherapy is assigned to a specific code of 92.24 (AHIMA 2012a, 677). 97. Identify the CPT procedure code(s) and modifier(s) if appropriate, for x-ray of the mandible, five views. a. 70100, 70110 b. 70100-50 c. 70110-50 d. 70110 - ✔✔Correct Answer: D Index Mandible, x-ray, resulting in code range 70100-70110. Review of the available codes indicates that code 70110 is the appropriate code when four or more views are obtained (AHIMA 2012a, 624). 98. An advantage of computer-assisted coding (CAC) is: a. Increased coding productivity b. Complexity, quality and format of health record documentation c. Technological limitations d. User resistance to change - ✔✔Correct Answer: A The AHIMA e-HIM Workgroup on Computer-Assisted Coding identified advantages and barriers to CAC. The advantages of CAC include increased coding productivity (Sayles and Trawick, 2010, 361). 99. The computer software program that assigns appropriate DRGs according to information provided for each episode of care is called a: a. Classification b. Database c. Diagnostic decision support (DDS) system d. Grouper - ✔✔Correct Answer: D Coders enter the codes that have been selected into a computer program called a grouper which assigns the patient's case to the correct group based on the ICD-9-CM and/or CPT/HCPCS codes (Johns 2011, 272). 100. Which of the following does not have to be included in a covered entity's notice of privacy practice? a. Description with one example of disclosures made for treatment, payment, and healthcare operations b. Description of all the other purposes for which a covered entity is permitted or required to disclose PHI without consent or authorization c. Statement of individual's rights with respect to PHI and how the individual can exercise these rights d. Signature of the patient and date the notice was given to the patient - ✔✔Correct Answer: D The NPP is a statement mandated by the HIPAA Privacy Rule issued by a healthcare organization that informs individuals of the uses and disclosures of patient-identifiable health information that may be made by the organization, as well as the individual's rights and the organization's legal duties with respect to that information (Brodnik et al. 2009, 249). 1. Identify the diagnosis code[s] for carcinoma in situ of vocal cord. a. 231.0 b. 161.0 c. 239.1 d. 212.1 - ✔✔a. 231.0 Index Carcinoma, in situ, see also Neoplasm, by site, in situ. 2. Identify the diagnosis code[s] for melanoma of skin of shoulder. 1. 172.8, 172.6 b. 172.6 c. 172.9 d. 172.8 - ✔✔b. 172.6 Index Melanoma [malignant], shoulder. Melanoma is considered a malignant neoplasm and is referenced as such in the index of ICD-9-CM. The term "benign neoplasm" is considered a growth that does not invade adjacent structures or spread to distant sties but may displace or exert pressure on adjacent structures . Which of the following organizations is responsible for updating the procedure classification of ICD-9- CM? a. Centers for Disease Control [CDC] b. Centers for Medicare and Medicaid Services [CMS] c. National Center for Health Statistics [NCHS] c. World Health Organization [WHO] - ✔✔b. Centers for Medicare and Medicaid Services [CMS] NCHS-Natl Center for Health Statistics is responsible for updating the diagnosis classification[Vol 1 & Vol 2], and CMS is responsible for updating the procedure classification [Vol 3]. 4. At which level of classification system are the most specific ICD-9-CM codes found? a. Category level b. Section level c. Subcategory level d. Subclassification level - ✔✔d. Subclassification level The most specific codes in the ICD-9-CM system are found at the subclassification level. 5. What are five-digit ICD-9-CM diagnosis codes referred to as? a. Category codes b. Section codes c. Subcategory codes d. Subclassification codes - ✔✔d. Subclassification codes Five-digit code numbers represent the subclassification level What are four-digit ICD-9-CM codes referred to as? a. Category codes b. Section codes c. Subcategory codes d. Subclassification codes - ✔✔c. Subcategory codes Categories are divided into subcategories. At this level, four-digit code numbers are used. Which of the following ICD-9-CM codes are always alphanumeric? a. Category codes b. Procedure codes c. Subcategory codes d. V codes - ✔✔d. V codes V codes are always alphanumeric codes. They are easy to identify because they begin with the alpa character V and follow with numeric digits. Which of the following ICD-9-CM codes classify environment events and circumstances as the cause of an injury, poisoning, or other adverse affect? a. Category codes b. E codes c. Subcategory codes d. V codes - ✔✔b. E codes E codes provide a means to describe environmental events, circumstances, and conditions as the cause of injury, poisoning, and other adverse effects. Which volume of ICD-9-CM contains the Tabular and Alphabetic Index of procedures? a. Volume 1 b. Volume 2 c. Volume 3 d. Volume 4 - ✔✔c. Volume 3 ICD-9-CM Volume 3 contains the Tabular List and Alphabetic Index of procedures. Identify the correct diagnosis code for lipoma of the face. a. 214.1 b. 213.0 c. 214.0 d. 214.9 - ✔✔c. 214.0 Index Lipoma, face. ICD-9-CM classifies neoplasms by system, organ, or site with the exception of neoplasms of the lymphatic and hematopoietic system, malignant melanomas of the skin, lipomas, common tumors of the bone, uterus, and ovary. Because of these exceptions, the Alphabetic Index must first be checked to determine whether a code has been assigned for that specific histology type. 11.** Identify the correct diagnosis code[s] for adenoma of adrenal cortex with Conn's syndrome. a. 227.0, 255.12 b. 227.0 c. 255.12 d. 225.12, 227.8 - ✔✔a. 227.0, 255.12 Index Adenoma, adrenal [cortex]. Index Syndrome, Conn. According to the Index in ICD-9-CM, except where otherwise indicated, the morphological varieties of adenoma should be coded by site as for "Neoplasm, benign". Which of the following is a standard terminology used to code medical procedures and services? a. CPT b. HCPCS c. ICD-9-CM d. SNOMED CT - ✔✔a. CPT CPT is a comprehensive descriptive listing of terms and codes for reporting diagnostic and therapeutic procedures and medical services. Identify the appropriate ICD-9-CM diagnosis code for cerebral contusion with brief loss of consciousness. a. 924.9 b. 851.42 c. 851.82 d. 851.81 - ✔✔c. 851.82 Index Contusion, cerebral-see Contusion, brain. Add a fifth digit of "2" for brief loss of consciousness. Cerebral contusions are often caused by a blow to the head. A cerebral contusion is a more severe injury involving a bruise of the brain with bleeding into the brain tissue, but without disruption of the brain's continuity. The loss of consciousness that occurs often lasts longer than that of a concussion. Codes for cerebral laceration and contusion range from 851.0 to 851.9, with fifth digits added to indicate whether a loss of consciousness or concussion occurred. 14. If a patient has an excision of a malignant lesion of the skin, the CPT code is determined by the body area from which the excision occurs and which of the following? a. Length of the lesion as described in the pathology report. b. Dimension of the specimen submitted as described in the pathology report. c. Width times the length of the lesion as described in the operative report. d. Diameter of the lesion as well as the most narrow margins required to adequately excise the lesion described in the operative report. - ✔✔d. Diameter of the lesion as well as the most narrow margins required to adequately excise the lesion described in the operative report. The code selection is determined by measuring the greatest clinical diameter of the apparent lesion plus that margin required for complete excision [lesion diameter plus narrow margins required equals the excised diameter]. 15. According to CPT, a repair of a laceration that includes retention sutures would be considered what type of closure? a. Complex b. Intermediate c. Not specified d. Simple - ✔✔a. Complex Complex closure includes the repair of wounds requiring more than layered closure, namely, scar revision, debridement, extensive undermining, stents, or retention sutures. 16. A patient is admitted with spotting. She had been treated two weeks previously for a miscarriage with sepsis. The sepsis had resolved, and she is afebrile at this time. She is treated with an aspiration dilation and curettage. Products of conception are found. Which of the following should be the principal diagnosis? a. Miscarriage b. Complications of spontaneous abortion with sepsis c. Sepsis d. Spontaneous abortion with sepsis - ✔✔a. Miscarriage Subsequent admissions for retained products of conception following a spontaneous or legally induced abortion are assigned the appropriate code from category 634, spontaneous abortion, or 635, legally induced abortion, with a fifth digit of "1" [incomplete]. This advice is appropriate even when the patient was discharged previously with a discharge diagnosis of complete abortion. 17. An 80-year-old is admitted with fever, lethargy, hypotension, tachycardia, oliguria, and elevated WBC. The patient has more than 100,000 organisms of Escherichia coli per cc of urine. The attending physician documents "urosepsis". How should the coder proceed to code this case? a. Code sepsis as the principal diagnosis with urinary tract infection due to E. coli as secondary diagnosis. b. Code urinary tract infection with sepsis as the principal diagnosis. c. Query the physician to ask if the patient has septicemia because of the symptomatology. d. Query the physician to ask if the patient had septic shock so that this may be used as the principal diagnosis. - ✔✔c. Query the physician to ask if the patient has septicemia because of the symptomatology. The term "urosepsis" is nonspecific term. If that is the only term documented, only code 599.0 should be assigned based on the default for the term in the ICD-9-CM index, in addition to the code for the causal organism, if known. Septicemia results from the entry of pathogens into the bloodstream. Symptoms include spiking fever, chills, and skin eruptions in the form of petechiae or purpura. Blood cultures are usually positive; however, a negative culture does not exclude the diagnosis of septicemia. Several other clinical indications symptomology could indicate the diagnosis of septicemia. Only the physician can diagnose the condition based on clinical indications. Query the physician when the diagnosis is not clear to the coder. A 65-year-old patient, with a history of lung cancer, is admitted to a healthcare facility with ataxia and syncope and a fractured arm as a result of falling. The patient undergoes a closed reduction of the fracture in the emergency department and undergoes a complete workup for metastatic carcinoma of the brain. The patient is found to have metastatic carcinoma of the lung to the brain and undergoes radiation therapy to the brain. Which of the following would be the principal diagnosis in this case? a. Ataxia b. Fractured arm c. Metastatic carcinoma of the brain d. Carcinoma of the lung - ✔✔c. Metastatic carcinoma of the brain If treatment is directed at the malignancy, designate the malignancy as the principal diagnosis. The only exception to this guideline is if a patient admission or encounter is solely for the administration of chemotherapy, immunotherapy, or radiation therapy, assign the appropriate V code as the first-listed or principal diagnosis and the diagnosis or problem for which the service is being performed as a secondary diagnosis. 19. A patient was admitted for abdominal pain with diarrhea and was diagnosed with infectious gastroenteritis. The patient also has angina and chronic obstructive pulmonary disease. Which of the following would be the correct coding and sequencing for this case? a. Abdominal pain; infectious gastroenteritis, chronic obstructive pulmonary disease; angina b. Infectious gastroenteritis; chronic obstructive pulmonary disease; angina c. Gastroenteritis; abdominal pain; angina d. Gastroenteritis; abdominal pain; diarrhea; chronic obstructive pulmonary disease; angina - ✔✔b. Infectious gastroenteritis; chronic obstructive pulmonary disease; angina Gastroenteritis is characterized by diarrhea, nausea, and vomiting, and abdominal cramps. Codes for symptoms, signs, and ill-defined condition from Chapter 16 of the CPT codebook are not to be used as the principal diagnosis when a related definitive diagnosis has been established. Patients can have several chronic conditions that coexists at the time of their hospital admission and quality as additional diagnosis such as COPD and angina. 20. A patient is admitted with a history of prostate cancer and with mental confusion. The patient completed radiation therapy for prostatic carcinoma three years ago and is status post a radical resection of the prostate. A CT scan of the brain during the current admission reveals metastasis. Which of the following is the correct coding and sequencing for the current hospital stay? a. Metastatic carcinoma of the brain; carcinoma of the prostate; mental confusion b. Mental confusion; history of carcinoma of the prostate; admission for chemotherapy c. Metastatic carcinoma of the brain, history of carcinoma of the prostate d. Carcinoma of the prostate; metastatic carcinoma to the brain - ✔✔c. Metastatic carcinoma of the brain, history of carcinoma of the prostate When a primary malignancy has been previously excised or eradicated from its site and there is no further treatment directed to the site and there is no evidence of any existing primary malignancy, a code from category V10, personal history of malignant neoplasm, should be used to indicate the former site of the malignancy. Any mention of extension, invasion, or metastatic to another site is coded as a secondary malignant neoplasm to that site. The secondary site may be the principal. with the V10 code used as a secondary code. 21. A patient is admitted with abdominal pain. The physician states that the discharge diagnosis is prancreatitis versus noncalculus cholecystitis. Both diagnoses are equally treated. The correct coding and sequencing for this case would be: a. Sequence either the pancreatitis or noncalculus cholecystitis as principal diagnosis b. Pancreatitis; noncalculus cholecystitis; abdominal pain c. Noncalculus cholecystitis; pancreatitis; abdominal pain d. Abdominal pain; pancreatitis; noncalculus cholecystitis - ✔✔a. Sequence either the pancreatitis or noncalculus cholecystitis as principal diagnosis In the unusual instance when two or more diagnoses equally meet the patients for principal diagnosis, as determined by the circumstances of admission, diagnostic workup, and the therapy provided, and the Alphabetic Index, Tabular List, or another coding guideline does not provide sequencing direction in such as cases, any one of the diagnoses may be sequenced first. According to the UHDDS, which of the following is the definition of "other diagnoses"? a. Is recorded in the patient record b. Is documented by the attending physician c. Receives clinical evaluation of therapeutic treatment or diagnostic procedures or extends the length of stay or increases nursing care and monitoring. d. Is documented by at least physicians and the nursing staff. - ✔✔c. Receives clinical evaluation of therapeutic treatment or diagnostic procedures or extends the length of stay or increases nursing care and monitoring. For reporting purposes the definition for "other diagnoses" is interpreted as additional conditions that affect patient care in terms of requiring: clinical evaluation, therapeutic treatment, diagnostic procedures, extended length of hospital stay, increased nursing care, and monitoring. 23. A 7-year-old patient was admitted to the emergency department for treatment of shortness of breath. The patient is given epinephrine and nebulizer treatments. The shortness of breath and wheezing are unabated following treatment. What diagnosis should be suspected? a. Acute bronchitis b. Acute bronchitis with chronic obstructive pulmonary disease. c. Asthma with status asthmaticus d. Chronic obstructive asthma - ✔✔c. Asthma with status asthmaticus A patient in status asthmaticus fails to respond to therapy administered during an asthmatic attack. This is a life-threatening condition that requires emergency care and likely hospitalization. 24. A patient is seen in the emergency department for chest pain. After evaluation of the patient it is suspected that the patient may have gastroesophageal reflux disease [GERD]. The final diagnosis was "Rule out chest pain versus GERD". The correct ICD-9-CM code is: a. V71.7, Admission for suspected cardiovascular condition b. 789.01 Esophageal pain c. 530.81 Gastrointestinal reflux d. 786.50 Chest pain NOS - ✔✔d. 786.50 Chest pain NOS Signs, symptoms, abnormal tests results, or other reasons for the outpatient visit are used when a physician qualifies a diagnostic statement as "rule out" or other similar terms indicating uncertainty. In the outpatient setting the condition qualified in that statement should not be coded as if its existed. Rather, the condition should be coded to the highest degree of certainty, such as the sign or symptom the patient exhibits. In this case, assign the code 786.50, Chest pain NOS. 25. A skin lesion is removed form a patient's cheek in the dermatologist's office. The dermatologist documents "skin lesion" in the health record. Before billing the pathology report returns with a diagnosis of basal cell carcinoma. Which of the following actions should be coding professional do for claim submission? a. Code skin lesion b. Code benign skin lesion c. Code basal cell carcinoma d. Query the dermatologist - ✔✔c. Code basal cell carcinoma For outpatient encounters for diagnostic tests that have been interpreted by a physician, and the final report is available at the time of coding, code any confirmed or definitive diagnosis[es] documented in the interpretation. Do not code related signs and symptoms as additional diagnosis. Note: This differs from the coding practice in the hospital inpatient setting regarding abnormal findings on test results. 26. An epidural was given during labor. Subsequently, it was determined that the patient would require a C-section for cephalopelvic disproportion because of obstructed labor. Assign the correct ICD-9-CM diagnosis and CPT anesthesia codes. [Modifiers are not used in the example]. a. 660.11, 653.41, 64479 b. 660.11, 653.01, 01961 c. 660.11, 653.41, 01967, 01968 d. 660.11, 653.91, 01996 - ✔✔c. 660.11, 653.41, 01967, 01968 The disproportion was specified as cephalopelvic; thus the correct ICD-9-CM code is 653.41. Two codes are required for anesthesia: one for the planned vaginal delivery [01967] and an add-on code [01968] to describe anesthesia for cesarean delivery following planned vaginal delivery converted to cesarean. An instructional not guides the coder to use 01968 and 01967. 27. Which of the following statements does not apply to ICD-9-CM? a. It can be used as the basis for epidemiological research. b. It can be used in the evaluation of medical care planning for healthcare delivery systems. c. It can be used to facilitate data storage and retrieval d. It can be used to collect data about nursing care. - ✔✔d. It can be used to collect data about nursing care. According to Central Office on ICD-9-CM, ICD-9-CM is not used to collect data about nursing care. 28. Which of the following is not one of the purposes of ICD-9-CM? a. Classification of morbidity for statistical purposes. b. Classification of mortality for statistical purposes c. Reporting of diagnoses by physicians d. Identification of the supplies, products, and services provided to patients. - ✔✔d. Identification of the supplies, products, and services provided to patients. [HCPCS] According to Central Office on ICD-9-CM, ICD-9-CM is not used to identify supplies, products, and services used by patients. 29. Which volume of ICD-9-CM contains the numerical listing of codes that represent diseases and injuries? a. Volume 1 [Tab] b. Volume 2 [Index} c. Volume 3 [Pcs] d. Volume 4 - ✔✔a. Volume 1 ICD-9-CM Volume 1 is known as the Tabular List and contains the numerical listing of codes that represents diseases and injuries. When coding benign neoplasm of the skin, the section noted here directs the coder to: 216 Benign Neoplasm of Skin [Category] Includes: Blue Nevus Dermatofibroma Hydrocystoma Pigmented Nevus Syringoadenoma Syringoma Excludes: Skin of genital organs [221.0-222.9] 216 Skin of lip Excludes Vermilion border of lip [210.0] 216 Eyelid, including canthus Excludes: Cartilage of eyelid [215.0] a. Use category 216 for syringoma b. Use category 216 for malignant melanoma. c. Use category 216 for malignant neoplasm of the bone. d. Use category 216 for malignant neoplasm of the skin. - ✔✔a. Use category 216 for syringoma Follow instructions under the main term in the Alphabetic Index. Instructions in the index should be followed when determining which column to use in the neoplasm table. In this example, malignant is not a choice in the Alphabetic Index shown. Benign in category 216 indicates all of the diagnosis codes in this category are benign. 31. A patient was discharged with the following diagnoses: "Cerebral occlusion, hemiparesis, and hypertension. The aphasia removed before the patient was discharged". Which of the following code assignments would be appropriate for this case? 342.90 Hemiparesis affecting unspecified side. 342.91 Hemiparesis affecting dominant side 342.92 Hemiparesis affecting nondominant side 434.90 Cerebral artery occlusion occlusion unspecified, without mention of cerebral infarction. 434.91 Cerebral artery occlusion unspecified with cerebral infarction 401 Hypertension 401.1 Malignant hypertension 401.9 Unspecified hypertension 428.0 Congestive heart failure. 784.3 Aphasia a. 434.91, 342.92, 784.3, 401 b. 434.90, 342.90, 784.3, 401.9 c. 434.90, 342.90, 401.9 d. 434.90, 342.90, 784.3, 401.0 - ✔✔b. 434.90, 342.90, 784.3, 401.9 Code 434.91 is assigned when the diagnosis states stroke, cerebrovascular, or cerebrovascular accident [CVA] without further specification. The health record should be reviewed to make sure nothing more specific is available. Conditions resulting from an acute cerebrovascular disease, such as aphasia or hemiplegia, should be coded as well. 32. A patient is admitted to the hospital with shortness of breath and congestive heart failure. The patient subsequently develops respiratory failure. The patient undergoes intubation with ventilator management. Which of the following would be the correct sequencing and coding of this case? a. Congestive heart failure, respiratory failure, ventilator management, intubation. b. Respiratory failure, intubation, ventilator management. c. Respiratory failure, congestive heart failure, intubation, ventilator management. d. Shortness of breath, congestive heart failure, respiratory failure, ventilator management. - ✔✔a. Congestive heart failure, respiratory failure, ventilator management, intubation. Acute respiratory failure, code 518.81, may be assigned as a principal or secondary diagnosis depending on the circumstances of the inpatient admission. Chapter-specific coding guidelines [obstetrics, poisoning, HIV, newborn] provide specific sequencing direction. Respiratory failure may be listed as a secondary diagnosis. If respiratory failure occurs after admission, it may be listed as a secondary diagnosis. 33. A physician correctly prescribes Coumadin. The patient takes the Coumadin as prescribed but develops hematuria as a result of taking the medication. Which of the following is the correct way to code this case? a. Poisoning due to Coumadin b. Unspecified adverse reaction to Coumadin. c. Hematuria; poisoning due to Coumadin d. Hematuria; adverse reaction to Coumadin. - ✔✔d. Hematuria; adverse reaction to Coumadin. Adverse affects can occur in situation in which medication is administered properly and prescribed correctly in both therapeutic and diagnostic procedures. An adverse effect can occur when everything is done correctly. The first-listed diagnosis is the manifestations or the nature of the adverse effect, such as the hematuria. Locate the drug in the Substance column of the Table of Drugs and Chemicals in the Alphabetic Index to Diseases. Select the E code for the drug from the Therapeutic Use column of the Table of Drugs and Chemicals. Use of the E code is mandatory when coding adverse effects. A patient is admitted for chest pain with cardiac dysrhythmia to Hospital A. The patient is found to have an acute inferior myocardial with atrial fibrillation. After the atrial fibrillation was controlled and the patient was stabilized, the patient was transferred to Hospital B for a CABG x3. Using the codes listed here, what are the appropriate ICD-9-CM codes and sequencing for both hospitalizations? 410.00 Myocardial infarction of anterolateral wall, episode unspecified. 410.01 Myocardial infarction of anterolateral wall, initial episode 410.40 Myocardial infarction of inferior wall, episode unspecified 410.41 Myocardial infarction of inferior wall, initial episode 410.42 Myocardial infarction of inferior wall, subsequent episode 427 -Cardiac dysrhythmias 427.3 --Atrial fibrillation and flutter 427.31 Atrial fibrillation 786.50 Chest pain, unspecified 36.13 Aortocoronary bypass of three coronary arteries a. Hospital A: 427, 786.50, 427.31, 410,91; Hospital B: 410.92, 36.13 b. Hospital A: 410.41, 427, 427.31; Hospital B: 410.42, 36.13 c. Hospital A: 410.41, 427.31; Hospital B: 410.41, 36.13 d. Hospital A: 410.41, 427.31, 786.50; Hospital B: 41042, 36.13 - ✔✔c. Hospital A: 410.41, 427.31; Hospital B: 410.41, 36.13 Use a fifth digit of "1" to designate the first episode of care [regardless of facility site] for a newly diagnosed myocardial infarction. The fifth digit "1" is assigned regardless of the number of times a patient may be transferred during the initial episode of care. 35. A patient is admitted to the hospital with abdominal pain. The principal diagnosis is cholecystitis. The patient also has a history of hypertension and diabetes. In the DRG prospective payment system, which of the following would determine the MDC assignment for this patient? a. Abdominal pain b. Cholecystitis c. Hypertension d. Diabetes - ✔✔b. Cholecystitis The principal diagnosis determines the MDC assignment. A patient was admitted to the hospital with symptoms of a stroke and secondary diagnoses of COPD and hypertension. The patient was subsequently discharged from the hospital with a principal diagnosis of cerebral vascular accident and secondary diagnoses of catheter-associated urinary tract infection, COPD, and hypertension. Which of the following diagnoses should not be tagged as POA? a. Catheter-associated urinary tract infection b. Cerebral vascular accident c. COPD d. Hypertension - ✔✔a. Catheter-associated urinary tract infection All claims involving inpatient admissions to general acute care hospitals or other facilities that are subject to law or regulation mandating collection of present on admission information . Present on admission [POA] is defined as present at the time the order for inpatient admission occurs. Conditions that develop during an outpatient encounter, including emergency department, observation, or outpatient surgery, are considered POA. Any condition that occurs after admission is not considered a POA condition. 37. Which of the following is a condition that arises during hospitalization? a. Case mix b. Complication c. Comorbidity d. Principal diagnosis - ✔✔b. Complication A complication is a secondary condition that arises during hospitalization and is thought to increase the length of stay by at least one day for approximately 75% of the patients. 38. A 65-year-old female was admitted to the hospital. She was diagnosed with septicemia secondary to Staphylococcus aureus and abdominal pain secondary to diverticulitis of the colon. What is the correct code assignment? a. 038.8, 562.11, 789.00 b. 038.11, 562 c. 038.9, 562.11. 041.11 d. 039.9, 562.11 - ✔✔b. 038.11, 562 Septicemia generally refers to a systemic disease associated with the presence of pathological microorganisms or toxins in the blood, which can include bacteria, viruses, fungi, or other organisms. Code 038.11 is assigned for septicemia with Staphylococcus aureus. Because abdominal pain is a symptom of diverticulosis, only diverticulitis of the colon [562.11] is coded. 39. Patient had carcinoma of the anterior bladder wall fulgurated three years ago. The patient returns yearly for a cystoscopy to recheck for bladder tumor. Patient is currently admitted for a routine check. A small recurring malignancy is found and fulgurated during the cystoscopy procedure. Which is the correct code assignment? a. 188.3, V10.51, 57.49, 57.32 b. 198.1, 57.49 c. 188.3, 57.49 d. 198.1, 188.3, 57.49 - ✔✔c. 188.3, 57.49 When the primary malignant neoplasm previously removed by surgery or eradicated by radiotherapy or chemotherapy recurs, the primary malignant code for the site is assigned, unless the Alphabetic Index directs otherwise. 40. A patient with a diagnosis of ventral hernia is admitted to undergo a laparotomy with ventral hernia repair. The patient undergoes a laparotomy and develops bradycardia. The operative site is closed without the repair of the hernia. Which is the correct code assignment. a. 553.20, 427.89, V64.3, 54.11 b. 553.20, 997.1, 427.89, 54.19 c. 553.20, 54.11 d. 553.20, 54.11, V64.3 - ✔✔a. 553.20, 427.89, V64.3, 54.11 The ventral hernia is coded as the primary or first-listed diagnosis. The repair of the hernia is not coded because it was not performed; however, code 54.11 is assigned to describe the extent of the procedure, which is an exploratory laparotomy. The V64.3 is coded to indicate the cancelled procedure. Code 427.89 is also used to describe the bradycardia that the patient develops during the procedure. 41. These codes are used to assign a diagnosis to a patient who is seeking health services but is not necessarily sick. a. E codes b. V codes c. M codes d. C codes - ✔✔b. V codes V codes are diagnosis codes and indicate a reason for healthcare encounter. Patient was admitted through the emergency department following a fall from a ladder while painting an interior room in his house. He had contusion of the scalp and face and an open fracture of the acetabulum. The fracture site was debrided and the fracture was reduced by open procedure with an external fixation device applied. Which is the correct code assignment? a. 808.1, E881.0, E849.0, 79.25, 78.15 b. 808.1, 920, E881.0, E849.0, E000.8, E013.9, 79.25, 78.15, 79.65 c. 808.0, E881.0, E000.8, E013.9, 79.35, 79.65 d. 808.1, E881.0, E849.0, E013.9, 79.25, 78.15, 79.65 - ✔✔b. 808.1, 920, E881.0, E849.0, E000.8, E013.9, 79.25, 78.15, 79.65 The fracture is the principal diagnosis, with the contusions as a secondary diagnosis. The fracture is what required in the most treatment. Procedures for the reduction, debridement, and external fixation device would all need to be coded. 43. Assign the correct CPT code for the following procedure: Revision of the pacemaker skin pocket. a. 33223 b. 33210 c. 33212 d. 33222 - ✔✔d. 33222 Begin with the term Revision: pacemaker site; chest. 44. Assign the correct CPT code for the following: A 58-year-old male was seen in the outpatient surgical center for an insertion of self-contained inflatable penile prosthesis for impotence. a. 54401 b. 54405 c. 54440 d. 54400 - ✔✔a. 54401 Code 54401 is correct because the prosthesis is self-contained. 45. Patient returns during a 90-day postoperative period from a ventral hernia repair, now complaining of eye pain. What modifier would a physician use with the Evaluation and Management code? a. -79, Unrelated procedure or service by the same physician during the postoperative period. b. -25, Significant, separately identifiable evaluation and management service by the same physician on the same day of the procedure or other service. c. -21, Prolonged evaluation and management services d. -24, Unrelated evaluation and management service by the same physician during a postoperative period. - ✔✔d. -24, Unrelated evaluation and management service by the same physician during a postoperative period. Modifier -24 is used for unrelated evaluation and management service by the same physician during a postoperative period. A patient is admitted to an acute-care hospital for acute intoxication and alcohol withdrawal syndrome due to chronic alcoholism. a. 291.8, 303.00 b. 303.00 c. 305.00 d. 291.81, 303.00 - ✔✔d. 291.81, 303.00 If the patient is admitted in withdrawal or if withdrawal develops after admission, the withdrawal code is designated as the principal diagnosis. The code for substance abuse or dependence is listed second. A 45-year-old is admitted for blood loss anemia due to dysfunctional uterine bleeding. a. 280.0, 626.8 b. 285.1, 626.8 c. 626.8, 280.0 d. 280.0, 218.9 - ✔✔a. 280.0, 626.8 The anemia would be sequenced first based on principal diagnosis guidelines. 48. Patient admitted with senile cataract, diabetes mellitus, and extracapsular cataract extraction with simultaneous insertion of intraocular lens. a. 366.10, 250.50, 13.59, 13.71 b. 250.00, 366.10 c. 250.00, 366.12 d. 366.10, 250.00, 13.59, 13.71 - ✔✔d. 366.10, 250.00, 13.59, 13.71 The patient was admitted for the senile cataract and the procedures were completed for the condition. This follows the UHDDS guidelines for principle diagnosis selection. There is also no causal relationship given between the diabetes and the cataract, so 250.50 would be incorrect. 49. A patient is admitted with acute exacerbation of COPD, chronic renal failure, and hypertension. a. 492.8, 496, 403.10,585.9 b. 492.8, 585.9, 401.9 c. 496, 585.9, 401.9 d. 491.21, 403.91, 585.9 - ✔✔d. 491.21, 403.91, 585.9 Patient was admitted for COPD, so this is listed as the principal diagnosis. Code 491.21 is used when the medical record [Show More]

Last updated: 1 year ago

Preview 1 out of 460 pages

Add to cart

Instant download

document-preview

Buy this document to get the full access instantly

Instant Download Access after purchase

Add to cart

Instant download

Reviews( 0 )

$16.00

Add to cart

Instant download

Can't find what you want? Try our AI powered Search

OR

REQUEST DOCUMENT
91
0

Document information


Connected school, study & course


About the document


Uploaded On

Mar 15, 2023

Number of pages

460

Written in

Seller


seller-icon
Topmark

Member since 1 year

66 Documents Sold


Additional information

This document has been written for:

Uploaded

Mar 15, 2023

Downloads

 0

Views

 91

Document Keyword Tags


$16.00
What is Browsegrades

In Browsegrades, a student can earn by offering help to other student. Students can help other students with materials by upploading their notes and earn money.

We are here to help

We're available through e-mail, Twitter, Facebook, and live chat.
 FAQ
 Questions? Leave a message!

Follow us on
 Twitter

Copyright © Browsegrades · High quality services·